Download as pdf or txt
Download as pdf or txt
You are on page 1of 83

PHYSICS 1

TABLE OF CONTENTS

Science Pocket Lesson 1 Measurements 1


Science Pocket Lesson 2 Conversion of Units 5
Worksheet 1 Measurements and Conversion of Units 10
Science Pocket Lesson 3 Scientific Notation 12
Science Pocket Lesson 4 Errors in Measurements 16
Worksheet 2 Scientific Notation and Errors in Measurement 20
Science Pocket Lesson 5 Vectors and Scalars 21
Science Pocket Lesson 6 Vector Addition 25
Worksheet 3 Vector Addition 31
Science Pocket Lesson 7 Kinematics: Describing Motion 33
Science Pocket Lesson 8 Kinematics: Uniform Accelerated Motion 38
Science Pocket Lesson 9 Free Fall 43
Science Pocket Lesson 10 Graphing Motion 49
Worksheet 4 Graphing Motion (Slope) 57
Worksheet 5 Graphing Motion (Area) 59
Science Pocket Lesson 11 Projectile Motion 60
Worksheet 6 Projectile Motion 66
Science Pocket Lesson 12 Forces: Causes of Motion 68
Science Pocket Lesson 13 Newton's Laws of Motion 72
Worksheet 7 Forces and Newton's Laws of Motion 76
Science Pocket Lesson 14 Circular Motion 78
Worksheet 8 Circular Motion 83

SAN PEDRO RELOCATION CENTER NATIONAL HIGH SCHOOL – SCIENCE DEPARTMENT © 2020
PHYSICS 1
Measurements

SPOT GOAL

 Identify the different types of quantities


 Identify the importance of Measurements in our daily
lives
 Apply measurements at home

Measurements are everywhere! We use measurements every time – when we


eat, drink, take a bath, cook, sleep, almost in every activity that we do, we use
measurements. Can you identify what these things measure?

SAN PEDRO RELOCATION CENTER NATIONAL HIGH SCHOOL – SCIENCE DEPARTMENT © 2020 1
INSTANT TASK

Now let’s try doing some measurements at home. Try the following
tasks and then answer the questions that follow. Write your answers
on the space provided.

1. Stand beside your mom or your dad and identify who is taller than the other.
2. Get 2 identical chairs. Put some things on the other chair like books, bags, etc.
and then push the chairs one at a time. Identify which one is more difficult to
push.
3. Get 2 empty bottles/containers with different shape. Fill them up with water and
identify which bottle/container took longer time to be filled up.

Task 1:

 Who is taller than the other? _____________


 What quantity did you measure? ______________
 How did you measure that quantity?
____________________________________________________________________

Task 2:

 Which chair is more difficult to push? ___________________________


 What quantity did you measure? ____________
 How did you measure that quantity?
____________________________________________________________________

Task 3:

 Which bottle/container took longer time to be filled up? ______________________


 What quantity did you measure? ____________
 How did you measure that quantity?
___________________________________________________________________

2
SAN PEDRO RELOCATION CENTER NATIONAL HIGH SCHOOL – SCIENCE DEPARTMENT © 2020
DIRECT TALK

What you did in the instant task is called measuring.


You compare quantities in order to get a measurement.

So we can say that measurement is a Comparison of an unknown quantity with some known
quantity of the same kind.
We already know that measurement involves PHYSICAL QUANTITES – any number that is
used to describe a physical phenomenon quantitatively. Physical quantities could either be
FUNDAMENTAL or DERIVED quantity. Fundamental Quantities are the basics quantities
while Derived quantities are taken from combining the fundamental quantities by multiplying
or dividing them to other quantities. There are SEVEN fundamental quantities – LENGTH,
TIME, MASS, AMOUNT OF SUBSTANCE, TEMPERATURE, ELECTRIC CURRENT and LUMINOUS
INTENSITY. Derived Quantities include FORCE, POWER, SPEED, ACCELERATION, WORK,
AREA, VOLUME, PRESSURE, and many more!
A number is not a measurement. It must have a UNIT in order for it to become a
measurement. Unit gives identity to numbers. 1, for example is just a number, but if we say
1 meter, it becomes a measurement because of the unit. In measuring, we use different
units but we have a STANDARD UNIT, we call it the SI SYSTEM OF UNITS or SI UNIT. (The
abbreviation SI comes from the system’s French name “Système International.”) Table 1.1
shows the SI unit for the fundamental quantities and some derived quantities.
In addition to these units we can also use other
FUNDAMENTAL QUANTITIES units such as centimeters, kilometers, etc.
QUANTITY UNIT SYMBOL centi- and kilo- are called PREFIXES that denote
LENGTH meter m various powers of ten. Some of the most
TIME second s frequently used prefixes for the various powers
MASS kilogram kg
of ten and their abbreviations are listed in
Table 1.2
AMOUNT OF
mole mol
SUBSTANCE
POWER PREFIX ABBREVIATION
TEMPERATURE kelvin K -24
10 yocto y
ELECTRIC CURRENT ampere A 10-21 zepto z
10-18 atto a
LUMINOUS INTENSITY candela cd
10-15 femto f
10-12 pico p
DERIVED QUANTITIES 10-9 nano n
10-6 micro 
QUANTITY UNIT SYMBOL
10-3 milli m
FORCE newton N
10-2 centi c
POWER watts W 10-1 deci d
SPEED meter per second m/s 101 deka da
3
meter per second 10 kilo k
ACCELERATION m/s2
squared 106 mega M
WORK joule J 109 giga G
1012 tera T
AREA square meter m2
1015 peta P
VOLUME cubic meter m3 1018 exa E
Table 1.1 Fundamental and Derived Units 1021 zetta Z
1024 yotta Y
Table 1.2 Prefixes for SI Units

SAN PEDRO RELOCATION CENTER NATIONAL HIGH SCHOOL – SCIENCE DEPARTMENT © 2020 3
FLASH CHECK Let’s go back
to the Start Up. The
Now, can you give me the importance of pictures there are:
measurement in the following scenarios? 1. Alarm clock which
measures TIME;
2. Measuring cups which
1. In baking cakes/cooking meals measures VOLUME of
____________________________________ liquids (oil, water,
____________________________________ milk) or solids (sugar,
flour); and
2. When doing an experiment at school 3. Weighing scale which
____________________________________ measures your
____________________________________ WEIGHT.

3. Paying the cashier in the grocery store


_______________________________________________________
_______________________________________________________
4. When taking a bath
_______________________________________________________
_______________________________________________________

QUICK REFERENCES
CONNECT https://www.ugr.es/~zoom/fermi/1%20%20
Physics%20and%20Measurement.pdf

https://www.nyu.edu/pages/mathmol/textb
ook/weightvmass.html#:~:text=1)%20Mass
%20is%20a%20measurement,is%20meas
MASS WEIGHT ured%20on%20a%20scale.
kilogram newton
Amount of Freedman, R.A. (2012). University Physics
matter Pull of gravity With Modern Physics. 13th Edition. SF
something on an object California: Smith
contains
Doesn’t change Changes with
with location of location of the
the object object

4
SAN PEDRO RELOCATION CENTER NATIONAL HIGH SCHOOL – SCIENCE DEPARTMENT © 2020
PHYSICS 1
Conversion of Units

SPOT GOAL

 Identify and apply the process/Steps in Conversion


of Units
 Cite importance of Conversion of Units in everyday
life

UNITS are very important when it comes to measurement. Even at home we could
encounter UNITS!
Below are some pictures of the things I have at home. Can you identify the units used
there?

SAN PEDRO RELOCATION CENTER NATIONAL HIGH SCHOOL – SCIENCE DEPARTMENT © 2020 5
INSTANT TASK

Try the following tasks and then answer the questions that follow.
Write your answers on the space provided.

1. Get a calendar. Count the number of days present in the month


of September.

2. Observe your clock; count the number of times the second


hand move for every one move of the minute hand.

3. Get a desk ruler, the one like that in the picture, the inch and cm ruler.
Identify the last numbers indicated in each scale.

Task 1:
There are _____ days in the month of September
Task 2:
For every one move of the minute hand the second hand moves ______
times
Task 3:
The last numbers for inches and centimeter scales are ________ and
________ respectively.

What are the units of measurement used in each task?


Task 1: ___________ and ____________
Task 2: ____________ and ____________
Task 3: ____________ and ____________

6
SAN PEDRO RELOCATION CENTER NATIONAL HIGH SCHOOL – SCIENCE DEPARTMENT © 2020
DIRECT TALK

What you did in the Instant Task is comparison of units.


We can see that for measuring TIME, even though the
SI Unit is second, we usually use months, days, minutes, and many more.
For length, even though the SI Unit is meter, we use inches and centimeter for a
desk ruler. Sometimes it is necessary to CONVERT units from one system to another.
When Converting Units, we only change the unit of measurement but the VALUE
remains the same. So we can say that 1 month has the same value as 30 days;1
minute has the same value as 60 seconds; and 12 inches has the same value as 30
centimeters. These are known as CONVERSION FACTORS. Some of the commonly
used conversion factors are listed on Table 2.1.

LENGTH The question now is HOW DO WE CONVERT UNITS?


1m = 100 cm = 1000 mm
1 km = 1000 m = 0.6214 mi The first step in unit conversion is to identify the given units
1m = 3.281 ft = 39.37 in and the desired units. Then, multiply the given units to the
1 cm = 0.3937 in appropriate conversion factor. The operation must be done in
1 in = 2.540 cm such a way that the units you want to get rid of (given units)
MASS are canceled and the units you want to end up with (desired
1 oz = 28.35 g units) remain. One thing to remember, Units obey the
rules of algebra.
VOLUME
1 liter = 1000 cm3 = 10 -3 m3
For example, suppose we wish to convert 15 in to cm. The
1 gallon = 3.788 liters
given unit is 15 in, and the desired unit is cm. The conversion
1 ml = 0.033814 fl oz
factor that we will be using is 1 in = 2.540 cm. We find that
TIME
1 min = 60 s 2.540 cm
1 h = 3600 s 15 in = 38.1 cm
1 in
1 d = 86,400 s
2.540 cm
Table 2.1 Unit Conversion This works because multiplying by is the same as
Factors 1 in
multiplying by 1, because the numerator and denominator
describe identical things.

Example 2.1 Converting speed units Solution:

The world land speed record is 763.0 mi/h, 1000 m 1h m


set on October 15, 1997, by Andy Green in 763.0 mi/h = 341.0
0.6214 mi 3600 s s
the jet-engine car Thrust SSC. Express this
speed in meters per second. So we can say that 763.0 mi/h = 341.0 m/s
We first Identify the given and desired units: This was the first supersonic land speed (the
Given unit = 763.0 mi/h
speed of sound in air is about 340 m/s)
Desired unit = m/s
Conversion Factors: Supersonic travel is rate of travel of an object that
1000 m = 0.6214 mi exceeds the speed of sound. Amazing!
1 h = 3600 s

7
SAN PEDRO RELOCATION CENTER NATIONAL HIGH SCHOOL – SCIENCE DEPARTMENT © 2020
Example 2.2 Converting Volume units
Solution:
The world’s largest cut diamond is the First
Star of Africa (mounted in the British Royal
Sceptre and kept in the Tower of London). Its 1 m3
1.84 in3 = 3.02 x 10−5 m3
volume is 1.84 cubic inches. What is the 61023.38 in3
volume in cubic meters?

We first Identify the given and desired units: So take note that if you encounter units with
Given unit = 1.84 in3 exponents make sure that you express the
Desired unit = m3 conversion factors in the same exponent as
Conversion Factors: well.
1 m = 39.37 in However, since the given and
desired units are in cubic, we need also to
cube the conversion factors:
1 m3 = 61023.38 in3

TRY THIS!
(A) The mass of a solid cube is 856 g, and each edge has a length of 5.35 cm. Determine the
density  of the cube in basic SI units. (the formula for Density = mass/volume)

Solve here:

Let’s go back to the Start Up. Kindly prove that the measurements in each picture are
equivalent.

Solve here:

Check your Answer: (A) 5.59 x 103 kg/m3

8
SAN PEDRO RELOCATION CENTER NATIONAL HIGH SCHOOL – SCIENCE DEPARTMENT © 2020
FLASH CHECK
Solve the problem below. Make sure to follow the steps
and box your final answer.

How many nanoseconds does it take light to travel 1.00 ft in vacuum?


(speed of light in vacuum c = 3.00 x 108 m/s)

Solve here:

QUICK REFERENCES
CONNECT https://www.ugr.es/~zoom/fermi/1%20%20Physi
cs%20and%20Measurement.pdf

https://phys.libretexts.org/Bookshelves/Universit
y_Physics/Book%3A_University_Physics_(Ope
Baking uses a lot of equipment – we
nStax)/Map%3A_University_Physics_I_-
have the oven, rolling pin, baking
sheets, baking pans, and oh! Don’t Mechanics%2C_Sound%2C_Oscillations%2C_
forget the measuring cups! and_Waves_(OpenStax)/01%3A_Units_and_M
We use measuring cups to measure easurement
the flour, the sugar, the baking
powder that we need in baking. But Freedman, R.A. (2012). University Physics With
it’s also used in measuring volume of Modern Physics. 13th Edition. SF California:
liquid like milk, all you have to do is
Smith
to convert cups to milliliters!

SAN PEDRO RELOCATION CENTER NATIONAL HIGH SCHOOL – SCIENCE DEPARTMENT © 2020 9
WORKSHEET 1 – MEASUREMENT AND CONVERSION OF UNITS

I. MEASURMENT: Encircle the correct units.

7:00 AM - I had a wash in 3 (SECONDS, LITERS, MILLILITER) of water.

7:30 AM - For breakfast I had 200 (KILOGRAMS, MILLIGRAMS, GRAMS) of cereal,


with 100 (GRAMS, MILLILITERS, LITERS) of milk. I also had a cup of tea, with ½ a
(GRAM, LITER, METER) of sugar in it.

8:10 AM - I had to run for the bus today. The bus stop is 100 (METERS, MILES,
MILLIMETERS) away from my house.

11:00 AM - During break I shared out a LITRE bottle of lemonade, between five glasses.
We each had 200 (LITERS, MILLILITERS, TONNES) of lemonade.

3:15 PM - This afternoon we had P.E. I kicked the football 14 (METERS,


CENTIMETERS, KILOMETERS).

4:30 PM - My brother is running in an Athletics competition this evening. He is running


in the 1 500 (METERS, KILOMETERS, MILLIMETERS) event. He can run that distance in 6
(MINUTES, HOURS, SECONDS). My brother is taller than I am. He is 1.82
(CENTIMETERS, KILOMETERS, METERS) tall.

7:30 PM - The road outside our house looks like a river! The main water pipe has burst
and there are (MILLILITERS, METERS, LITERS) of water gushing down the road.

7:45 PM - I am going to watch a film on TV. It is 1 ½ (MINUTES, HOURS,


SECONDS) long.

9:00 PM - I have just weighed my pet hamster, Gerald. Well, I tried to weigh him, but
he jumped off the scales just as the arrow reached 250 (KILOGRAMS, GRAMS,
MILLIGRAMS).

9:30 PM - Time for bed! I have to get up in 10 (METERS, MINUTES, HOURS). Before
I went to sleep I had 250 (MILLILITERS, MILLIGRAMS, MILLIMETERS) of tea to drink.

II. Read and Answer the following questions.

1. For an answer to be complete, the units need to be specified. Why?


_______________________________________________________________________
_______________________________________________________________________

2. Write the following as full (decimal) numbers without prefixes on the units: (a) 286.6
mm, (b) 85 m, (c) 62.1 ps, and (d) 22.5 nm.

(a) __________ (b) __________ (c) __________ (d) __________

3. Express the following using the prefixes of Table 1.2: (a) 1 x 106 volts, (b) 2 x 10 -6

meters, (c) 6 x 10 3 days, and (d) 7 x 10 -9 seconds.

(a) __________ (b) __________ (c) __________ (d) __________

10 SAN PEDRO RELOCATION CENTER NATIONAL HIGH SCHOOL – SCIENCE DEPARTMENT © 2020
III. Solve the following conversion problems. Show your complete solution on
the box provided.

1. The Sun, on average, is 93 million miles from Earth. How many meters is this?

8
2. A light-year is the distance light travels in one year (at speed = 2.998 x 10 m/s). How
many meters are there in a light-year?

3. American football uses a field that is 100.0 yd long, whereas a soccer field is 100.0 m
long. Which field is longer? (1 yd = 0.9144 m)

REFERENCES:
https://prep.bishops.org.za/gr4/2010/Maths/term%203%20wksheets%202010.pdf
Giancoli, D.C (2014). Physics, Principles with Applications. 7th Edition. USA. Smith

SAN PEDRO RELOCATION CENTER NATIONAL HIGH SCHOOL – SCIENCE DEPARTMENT © 2020
11
PHYSICS 1
Scientific Notation

INSTANT TASK

There are a lot of things that we can find in our home. Some of these give a

VERY LARGE MEASUREMENT and some give VERY SMALL

Can you list 5 things that give very large and very small
MEASUREMENTS.

measurements that you can find in your house/surroundings? Write your answers
below.

VERY LARGE VERY SMALL MEASUREMENTS

MEASUREMENT
Example: Mass of the Sun The mass of a grain of salt in kilograms

12
SAN PEDRO RELOCATION CENTER NATIONAL HIGH SCHOOL – SCIENCE DEPARTMENT © 2020
DIRECT TALK

There are some disadvantages when dealing with very large and very
small measurements. First is that it is prone to Uncertainty/Errors
and the other is that writing these measurements brings inconvenience.
So, Scientists invented a method of writing very large and very small numbers in a
convenient form that we now know as SCIENTIFIC NOTATION.

A number is written in scientific notation when it is expressed in the form:


Where:

a is the coefficient that is greater than


n
a x 10 or equal to 1 but less than 10.
n is the positive (+) or negative (-)
exponent

Now the Question is HOW DO WE EXPRESS A MEASUREMENT IN SCIENTIFIC NOTATION?

Here are the STEPS:


1. Locate the decimal point
2. Move the decimal point either to the LEFT or to the RIGHT as long as the final number will
be in between 1 to 10. (Greater than or equal to 1 but less than 10). The number of
movement of the decimal point indicates the exponent.
3. If you move the decimal point to the LEFT , the exponent is always positive (+), if
you move it to the RIGHT , it is negative (-)
4. Then rewrite it in Scientific Notation form.

Example 3.1 Example 3.2


Express 16,000,000,000 m in Scientific Express 0.0000000016 m in Scientific
Notation Notation

Solution Solution
1. Locate the decimal point: 1. Locate the decimal point:
16,000,000,000. m 0.0000000016 m

2. Move the decimal point to the LEFT ten 2. Move the decimal point to the RIGHT
times in order to obtain a number between nine times in order to obtain a number
1 – 10: between 1 – 10:
1. 6 0 0 0 0 0 0 0 0 0. 0 . 000000001 .6m

10
3. Rewrite: 1.6 x 10 m 3. Rewrite: 1.6 x 10 m
-9

SAN PEDRO RELOCATION CENTER NATIONAL HIGH SCHOOL – SCIENCE DEPARTMENT © 2020 13
What if we are given a number in scientific notation and would like to know how to
write it out (expand the number)?

Here are the STEPS:


1. If the exponent is POSITIVE, always move the decimal point to the RIGHT; if
it is NEGATIVE, always move the decimal point to the LEFT.
2. The number of movements is based on the exponent.
3. Fill in the spaces with zeros.
4. Rewrite the measurement.

Example 3.3 Example 3.4

Expand 4.5 x 106 m Expand 4.5 x 10-6 m

Solution Solution

The exponent is positive 6, therefore move The exponent is negative 6, therefore move
the decimal point to the right 6 times and the decimal point to the left 6 times and then
then fill in the spaces with 0. fill in the spaces with 0.

4 . 5 . 4.500000 . 0. 4.5 0.000004.5

Rewrite: 4,500,000 m Rewrite: 0.0000045 m

TRY THIS! Let’s go back to START UP!


(A) Rewrite the following numbers in scientific notation
Scientists use Light-years to
1. 243,000,000 describe the distances of the
space object. This is because
2. 0.000 000 586 most objects in space are
reeeaaaaaaaalllly far away
3. 9,100 from each other. Imagine, one
light-year is about 9 trillion km
or 9,000,000,000,000 km.
4. 0.000 5
that’s just 1 light-year!

(B) Expand the following numbers using any method.


Aside from Scientific notation, we
also have ENGINEERING
1. 4.8 x 104 NOTATION. It is also written in
the same form. However, the
2. 5.6 x 10-2 coefficient doesn’t need to be
between 1-10 but the exponent
3. 9.01 x 10-8 must be divisible by 3.
Ex. 45 x 1012, 125 x 106
4. 8.53 x 107

0.0000000901; 4) 85,300,000
Check your answers: (A) 1) 2.34 x 108; 2) 5.86 x 10-7; 3) 9.1 x 103; 4) 5.0 x10-4; (B) 1) 48,000; 2) 0.056; 3)

14
SAN PEDRO RELOCATION CENTER NATIONAL HIGH SCHOOL – SCIENCE DEPARTMENT © 2020
FLASH CHECK
Let’s go back to INSTANT TASK. Below are some of the things that can be found
at home/the environment that have very large/small measurements. Express these
measurements in scientific notation or expand them to their original form.

Mass of grain of salt:


0.00000005850 kg
Diameter of strand of hair:
0.000181 m
Mass of the Sun:
Speed of Light: 1.989 × 1030 kg
3 x 108 m/s

QUICK REFERENCES
CONNECT https://www.ohlone.edu/mathmods/mathmod1
https://www.dreamstime.com/illustration/doodle-salt.html
https://www.bluebulbprojects.com/MeasureOfThings/results.p
Philippines is one of the many hp?comp=weight&unit=kgms&amt=3.0E-
7&sort=pr&p=1#:~:text=The%20mass%20of%20a%20grain%
countries that experienced the
20of%20salt%20is%20approximately%200.00000005850%2
Corona Virus Disease 2019 (COVID - 0kilograms
19) pandemic. Because of that, the https://www.123rf.com/clipart-
Government decided that the nation vector/strand_of_hair.html?sti=n0qoxzt6tai0h6dnrd|
will be placed in an Enhanced https://hypertextbook.com/facts/1999/BrianLey.shtml#:~:text=
Community Quarantine (ECQ). It As%20a%20person%20grows%20up,(millionths%20of%20a
%20meter).
started on March 17, 2020 and ended https://clipartlook.com/img-90575.html
on May 31, 2020. That’s 7.5 x 101 in hhttps://www.pinterest.ph/pin/107242034863894087/ttps://en.
days; 1.8 x 103 in hours; 1.08 x 105 in wikipedia.org/wiki/Sun
min and 6.48 x 106 in seconds. That’s https://www.freepik.com/premium-vector/word-design-stop-
the time we spent together with our spread_7567587.htm
families while staying at home! https://pngtree.com/freepng/hand-drawn-new-coronavirus-
2019-ncov-banned-map_5329736.html
http://justice-everywhere.org/international/philosophers-
rundown-on-the-coronavirus-crisis/
https://spaceplace.nasa.gov/light-year/en/

SAN PEDRO RELOCATION CENTER NATIONAL HIGH SCHOOL – SCIENCE DEPARTMENT © 2020 15
PHYSICS 1
Errors in Measurements

INSTANT TASK

FIND THAT WORD!


Encircle the words in the puzzle that is related to the word MEASUREMENTS.

16 SAN PEDRO RELOCATION CENTER NATIONAL HIGH SCHOOL – SCIENCE DEPARTMENT © 2020
DIRECT TALK

These are the words that can be found from the INSTANT TASK:
Data, Accurate, Precise, Errors, Magnitude, Numbers, Quantity, Unit,
Systematic, and Experiment. These words are related to measurements
and for this lesson we will discuss the Errors in measurement also known as
uncertainties and the precision and accuracy of a measurement.

There are no perfect measurements; only Accurate and Precise measurements. This
is because of the errors in measurements. When we say ACCURACY, we mean how
close a measurement is to its true value. A true value is the actual measurement,
the standard. When we say PRECISION on the other hand, we mean the
repeatability of a measurement using a given instrument , meaning you get the
same/closely the same measurements after you do some trials. So again, we often
get measurements that are either accurate or precise but not perfect and that is
because of errors in a measurement.

When we say ERRORS, we mean the maximum difference between the measured
value and the true value. We have 2 kinds of Errors: SYSTEMATIC ERROR and
RANDOM ERROR.

SYSTEMATIC ERROR usually comes from the measuring instruments. They may occur
because there is something wrong with the instrument, probably it is not correctly
calibrated, or its data handling system, or because the instrument is wrongly used by
the experimenter. It primarily influences a measurement's accuracy.
RANDOM ERROR, on the other hand, is caused by unknown and unpredictable
changes in the experiment. These changes may occur in the measuring instruments
or in the environmental conditions. It primarily affects precision.

These are some examples of Random Error:


 When taking a volume reading in a flask, you may read the value from a
different angle each time.
 Measuring your height is affected by minor posture changes.

These are some examples of Systematic Errors:


 Measuring length with a metal ruler will give a different result at a cold
temperature than at a hot temperature, due to thermal expansion of the
material.
 An improperly calibrated thermometer may give accurate readings within a
certain temperature range, but become inaccurate at higher or lower
temperatures.

Another thing to bear in mind, Random errors are essentially unavoidable, while
systematic errors are not. Some of the ways to avoid Systematic error is to make
sure that the instrument is properly calibrated and ensure that you know how to
use the instrument.

SAN PEDRO RELOCATION CENTER NATIONAL HIGH SCHOOL – SCIENCE DEPARTMENT © 2020 17
Let us go back to START UP.

The square is to help you make more shots. It serves as the standard for making a
shot. If the ball doesn’t touch that square, it is more likely for the ball not to enter
the ring.

TRY THIS!

(A) Identify which is a Random Error and which is a Systematic Error.

1. Forgetting to tare or zero a balance _________________


2. When weighing yourself on a scale, you position yourself slightly differently
each time ____________________
3. Measuring wind velocity depends on the height and time at which a
measurement is taken ___________________
4. Not reading the meniscus at eye level for a volume measurement
______________________

(B) Look at each target and decide whether the “hits” are accurate, precise, both
accurate and precise, or neither accurate nor precise:

1 2

and precise
Check your answers: (A) (1) Systematic Error (2) Random Error (3) Random Error) (4) Systematic Error (B) (1) Precise but not Accurate (2) Accurate

18
SAN PEDRO RELOCATION CENTER NATIONAL HIGH SCHOOL – SCIENCE DEPARTMENT © 2020
FLASH CHECK
(A)Identify which is a Random Error and which is a Systematic Error and Explain
why.
1. Measuring Reaction time; you may react too early or too late by different
amounts of time

2. Measured distance is different using a new cloth measuring tape versus an


older, stretched one

(B) How well do the measurements on the target represent:


In terms of Accuracy: ___________________________
_____________________________________________

In terms of Precision: ___________________________


_____________________________________________

QUICK REFERENCES
CONNECT https://www.thoughtco.com/random-vs-
systematic-error-4175358
When do we experience ERRORS at home?
https://www.quora.com/Why-do-basketball-
We usually experience ERRORS in board-have-a-square-drawn-close-to-the-
cooking. Ever heard of maalat or basket#:~:text=This%20square%20is%20to%2
matabang? We say those words
because we have a standard taste 0help,it%20is%20plain%20and%20simple.&tex
for that kind of dish. So if we say that t=How%20can%20you%20get%20the%20best
a dish is maalat/matabang, that %20portable%20basketball%20hoop%3F
means our measurement of the
seasoning is not accurate. If your dish https://www.haystack.mit.edu/edu/pcr/Data/pdf/
is ALWAYS maalat/matabang, then Worksheet-Accuracy%20and%20Precision-
we can say that your cooking is Final.pdf
precise and not accurate. But if your
dish is masarap always, then your https://www.nicepng.com/ourpic/u2q8a9a9e6o
cooking is accurate and precise with 0o0t4_basketball-backboard-net-stock-
less errors.
photography-clip-art-backboard/

http://search.coolclips.com/m/vector/vc063295/
cooking-supplies/

SAN PEDRO RELOCATION CENTER NATIONAL HIGH SCHOOL – SCIENCE DEPARTMENT © 2020 19
WORKSHEET 2 – SCIENTIFIC NOTATION AND ERRORS IN MEASUREMENT

I. Rewrite the following numbers in scientific notation

1) 1,085,909.56 _________________________
2) 0.0006745 _________________________
3) 0.000000675342 _________________________
4) 143.8909 _________________________
5) 9.6 _________________________
6) 0.4 _________________________
7) 0.00080897 _________________________
8) 675,438,291,467.07 _________________________

II. Expand the following scientific notation

1) 4.78 x 103 _________________________


2) 8.738 x 10-9 _________________________
3) 5.89763 x 10-1 _________________________
4) 5.6 x 100 _________________________
5) 7.98635 x 10-6 _________________________
6) 9.003457 x 1012 _________________________
7) 7.094 x 101 _________________________
8) 1.00076 x 10-5 _________________________

III. Bulls eye to win

Jamal, David, Marie and Helen spent the afternoon playing darts. In the last round they set
the target as the bulls eye. Each person was allowed seven throws. The results of their
game are shown below.
Questions
1. Who won the game of darts?
2. What did the players choose to have
as their reference value?
3. Whose game would you describe as:
(a) Precise but inaccurate (b)
Imprecise but accurate (c) Precise
and accurate (d) Imprecise and
inaccurate?
4. Who do you think needs to improve
their game to avoid experiencing the
same systematic error next time
they play?
5. What advice would you give that
person?

Answer here:
1. ___________________
2. ___________________
3. (a) _______________ (b) _______________ (c) _______________ (d) _______________
4. ___________________
5. ___________________________________________________________________________
___________________________________________________________________________

REFERENCES:
https://edu.rsc.org/download?ac=13785

20 SAN PEDRO RELOCATION CENTER NATIONAL HIGH SCHOOL – SCIENCE DEPARTMENT © 2020
PHYSICS 1
Vectors and Scalars

INSTANT TASK

Directions are very important in our everyday life. It tells someone how to do
something. It puts plans into actions. But in Physics, we deal ‘directions’ in a
different way. It is defined as the path that must be taken to reach a specific
place. Examples of Directions include North, South, East, West, up, down, left,
right and many more!

Below are some situations that may/may not include directions. Can you identify
which one involves directions? Do that by encircling the whole sentence.

A hockey player
skating at 15 mph
towards the goal.
A box on the floor
has a volume of 10
cubic feet

The temperature
outside is 15 °C.

A car is speeding
south along a
highway at 75 mph.

SAN PEDRO RELOCATION CENTER NATIONAL HIGH SCHOOL – SCIENCE DEPARTMENT © 2020 21
DIRECT TALK

Just like what I said earlier in the INSTANT TASK, Directions are
important to us. Imagine a world without directions and it will become a
mess. In Physics, we also give the same importance to directions.
As a matter of fact, Physics identified magnitudes having directions as VECTORS.
The ones that don’t have directions we call them SCALARS. Examples of scalar quantities are,
time, volume, and temperature. They don’t have directions. You don’t say 3 seconds to the
left or 25 cubic cm south or 5 °C North.
Usually, we use directions as a component of motion, so we will be using directions when
talking about measurements that describe motion. Such measurements that describe motion
include speed, distance, acceleration, force, momentum, and many more but not all of these
measurements are considered as VECTOR quantities. Speed and distance are not vector
quantities because they don’t have directions. But if we include directions such as North,
South etc., we can now say that they are VECTORS. Hence, they will be known as velocity
and displacement, respectively.

How do we represent a vector and a scalar quantity?


Let’s start with the scalar quantity. Distance is an example of scalar quantity. We use the
symbol, d, when writing a measurement for distance. Example: d = 5 meters. So, when
representing a scalar quantity, we just write the symbol as it is.

We have, however, a different way in representing a vector quantity.

1. GRAPHICAL – We use ARROWS. The tail


represents the origin, the length represents
Tail
the magnitude and the head represents the
direction.

2. SYMBOLS – when writing a quantity, we can identify if the quantity is a vector or a scalar
by how its symbol is written. If the symbol is written as it is, we say that it is a scalar
quantity. ‘d’ is the symbol for distance (a scalar quantity). If we include directions to
distance it becomes a vector quantity. Below are the ways to write displacement.

Boldface With arrow above it Boldface, italic, with arrow


above it

d d d
In our discussions, we will be writing vectors by simply putting an arrow above the symbol.

TRY THIS!
TRY THIS!
Write the vectors Force, Acceleration and Velocity.

1. Force 2. Acceleration 3. Velocity

3. 2. Check your answers: 1.

22
SAN PEDRO RELOCATION CENTER NATIONAL HIGH SCHOOL – SCIENCE DEPARTMENT © 2020
Other things to remember about vectors:

Vector and vector are EQUAL because


they have the same length and direction.

= =- Vector has the same magnitude as but


opposite direction; is the negative of .
These vectors are ANTIPARALLEL.
Figure 5.1 The meaning of vectors that have the same
magnitude and the same or opposite direction

Vectors having the same directions are considered as PARALLEL


as in Figure 5.2.

Figure 5.2 Parallel vectors

Vectors having 90° degree angles are said to be


PERPENDICULAR or ORTHOGONAL as in Figure 5.3.

Figure 5.3 Orthogonal vectors

A
TRY THIS!
B
Look at the map and identify which vectors
are:
C
1. Parallel: _______________
2. Antiparallel: _______________
3. Orthogonal: _________________ D E
4. Equal: _________________

Check your answers: 1. Vectors A and C; 2. Vectors C and E and vectors A and E; 3. Vectors A and B and vectors B and C; 4. Vectors A and C

SAN PEDRO RELOCATION CENTER NATIONAL HIGH SCHOOL – SCIENCE DEPARTMENT © 2020 23
FLASH CHECK
Let’s check your understanding! Answer the following:

1. Differentiate Scalar quantities from Vector quantities.

2. Give 2 examples each for vector and scalar quantity that are not yet given in
the discussion. Write the name of the quantity.

SCALAR VECTOR

3. Represent your given vectors in #2 using symbols.

QUICK REFERENCES
CONNECT https://www.istockphoto.com/vector/businessman-
figure-clip-art-next-to-an-information-sign-with-
arrows-which-shows-gm1164219870-319945183

In the Philippines, we can see a lot of road https://www.yourdictionary.com/direction


signs that guides the motorist as they travel. In https://www.coolkidfacts.com/scalars-vectors/
the START UP we can see 2 examples of http://clipart-library.com/free/field-hockey-player-
road signs with arrows. We discussed that
silhouette.html
arrows represent vectors and in vectors we
have directions. However, in these road signs, https://commons.wikimedia.org/wiki/File:Open_card

The arrows here represent direction(s) in board_box_husky.png


which the motorists are obliged to https://toppng.com/thermometer-weather-icon-png-
follow.
clip-art-weather-thermometer-PNG-free-PNG-
Images_205396
https://www.netclipart.com/isee/iboomi_moving-fast-
clipart-moving-fast-clip-art-images/
https://www.lto.gov.ph/images/Advisory/road-traffic-
signs-pavement-markings_v2.pdf

24 SAN PEDRO RELOCATION CENTER NATIONAL HIGH SCHOOL – SCIENCE DEPARTMENT © 2020
PHYSICS 1
Vector Addition

SPOT GOAL

 Identify the ways in adding Vectors


 Rewrite a vector in component form
 Perform Vector Addition

SAN PEDRO RELOCATION CENTER NATIONAL HIGH SCHOOL – SCIENCE DEPARTMENT © 2020 25
INSTANT TASK

Get a ruler and do the following.

1. Measure the length, in cm, of the following arrows. Estimate your answers in whole
numbers.
Measurements:
Arrow A: ____________
A
Arrow B: ____________
Arrow AB: ____________
B

AB

2. Measure the length, in cm, of the following arrows. Estimate your answers in whole
numbers.
Measurements:

C Arrow C: ____________
Arrow D: ____________
D
3. Measure the part of the arrows that did not overlap.

Measurements:
CD Arrow CD: ____________

4. Measure the length, in cm, of the following arrows. Estimate your answers in whole
numbers. Then draw an arrow from the head of arrow F to the tail of arrow E. Measure
the length of the arrow that you drew.

Measurements:
E Arrow E: ____________
Arrow F: ____________
Arrow EF: ____________

26
SAN PEDRO RELOCATION CENTER NATIONAL HIGH SCHOOL – SCIENCE DEPARTMENT © 2020
DIRECT TALK

What you did in the INSTANT TASK is called graphical method of adding
vectors. It’s called tail-head method because all that’s done there is connecting
the tail of one arrow to the head of the other arrow and then measure the length.
So the lengths there signify the magnitude of the vector. And for the direction, you can use a
protractor for that since angles are also considered as directions.
However, in our discussion we will be focusing on the analytical method of adding vectors.
Instead of using rulers and protractors we will use algebra to get the Resultant Vector.
RESULTANT VECTOR , also known as VECTOR SUM, is the result of adding two or more
vectors. The process for getting the Resultant vector is known as VECTOR ADDITION or
COMPOSITION OF VECTORS.

Now, how do we add vectors?


1. Same Direction: if two vectors have the same direction, just add the magnitudes and
then copy the direction

Example 6.1 Solution:

= 5 m, East The magnitudes are 5 m and 10 m. Adding them will give you 15
m. Therefore, the resultant vector is 15 m, East.
= 10 m, East
= 15 m, East

2. Opposite Direction: if two vectors have opposite direction, you replace the directions
with positive or negative signs based on the Cartesian plane.
And then perform the operation. The final sign will indicate the
direction.

Example 6.2

= 5 m, East

= 10 m, West
Solution:

The magnitudes are 5 m and 10 m. We cannot add


them because their directions are opposite.
Based on the Cartesian plane, East is positive and West is negative.
Performing the operation we will have:

= + = +5m + (-10 m) = -5 m.

The negative there indicates the direction. Therefore, the resultant


vector is 5 m, West.

= 5 m, West.

SAN PEDRO RELOCATION CENTER NATIONAL HIGH SCHOOL – SCIENCE DEPARTMENT © 2020 27
3. Perpendicular Direction: If the vectors are in perpendicular direction, use the
Pythagorean Theorem to get the resultant vector. For the direction, use the
trigonometric function tangent.

TRIGONOMETRIC FUNCTIONS:

SOH – CAH – TOA PYTHAGOREAN THEOREM:


c
a
sin = opp/hyp c2 = a2 + b2

 cos  = adj/hyp
tan  = opp/adj
b

Example 6.3 Solution:

= 5 m , East Resultant Vector


Draw first the vectors:
= 10 m, North

Then, use the Pythagorean Theorem to get the resultant vector.

c2 = a2 + b2
c = (5 m)2 + (10 m)2
2

c2 = 25 m2 + 100 m2
c2 = 125 m2
𝐜 𝟐 = 𝟏𝟐𝟓 𝐦𝟐
c = 11.18 m

For the angle, use the Use the inverse tangent to get the angle. In your
trigonometric function tangent: calculators look for the sign tan-l
𝐨𝐩𝐩𝐨𝐬𝐢𝐭𝐞 𝐬𝐢𝐝𝐞
tan  =  = tan-l(2)
𝐚𝐝𝐣𝐚𝐜𝐞𝐧𝐭 𝐬𝐢𝐝𝐞

𝟏𝟎 𝐦  = 63.43°
tan  =
𝟓𝐦
The resultant vector is 11.18 m, 63°
tan  = 2 m

TRY THIS! Solve here:

Find the resultant


vector for the
following vectors:

= 7 m, North

= 8 m, East

Check your answers: 3.26 m, 41°

28
SAN PEDRO RELOCATION CENTER NATIONAL HIGH SCHOOL – SCIENCE DEPARTMENT © 2020
4. Component Method: If you have 2 or more vectors and the angles are not equal to 90°.
But before that, let’s discuss VECTOR RESOLUTION.

VECTOR RESOLUTION is a process where one vector is broken down into two or more smaller
vectors. These smaller vectors are known as VECTOR COMPONENTS and are usually the x- and y-
component. Remember the SOH CAH TOA? We will use the sine and the cosine to get these
components.
In Vector resolution instead of finding the resultant vector, we are now given with the resultant
vector. Sides a and b are missing and the hypotenuse(c) is given. It’s like the opposite of Vector
Addition.

For finding the a: (in this case, the y-component)

a = c sin 
a c
For finding the b: (in this case, the x-component)

b = c cos 
b

Example 6.4
Find the Resultant vector for the following vectors: 3 N, 0°; 4 N, 90°; and 5 N, 217°

Solution:
First make a table for the x- and y-components of each given vectors. Then perform vector
resolution for each of the given vectors.

Given Vectors (c) x-component (b = c cos ) y-component (a = c sin )


3 N, 0° 3 N, East 0 (since 0° means East)
4 N, 90° 0 (since 90° means North) 4 N, North
5 N, 217° -3.99 N (negative means west) -3.01 N (negative means south)
Summation of x- and y-
Rx = -0.99 N or 0.99 N, West Ry = 0.99 N or 0.99 N, North
components

Now use the Pythagorean Theorem to get the resultant vector and the tangent function for the
angle.
R2 = Rx2 + Ry2 For the angle: tan  = 1 N
R 2 = (0.99 N)2 + (0.99 N)2 𝐨𝐩𝐩𝐨𝐬𝐢𝐭𝐞 𝐬𝐢𝐝𝐞
 = tan-l(1)
tan  =
2 2 2 𝐚𝐝𝐣𝐚𝐜𝐞𝐧𝐭 𝐬𝐢𝐝𝐞
R = 0.98 N + 0.98 N
R2 = 125 m2 𝟎.𝟗𝟗 𝐍  = 45°, but since the vector lies
tan  =
𝟎.𝟗𝟗 𝐍 on the second quadrant, we
𝐑𝟐 = 𝟏. 𝟗𝟔 𝐍 𝟐
must add 90 to the angle.
R = 1.40 N
R = 1.40 N, 135°

TRY THIS!

Find the resultant vector for the following vectors: 3.76 cm, 34.5° and 4.53 cm, 34.1°

Final Answer:

Check your answer: 4.72 cm, 82°

SAN PEDRO RELOCATION CENTER NATIONAL HIGH SCHOOL – SCIENCE DEPARTMENT © 2020 29
FLASH CHECK
Find the resultant vector for the following vectors: 36 m, 34° and 23 m, 116°.
Answer Here:

QUICK REFERENCES
CONNECT https://postboxmap.com/en
https://en.wikipedia.org/wiki/Origin_(mathemati
Ever heard of SHORT- CUTS when going somewhere?
We say “may short-cut dito” when we mean shorter cs)
path towards our destination. https://study.com/academy/lesson/defining-
negative-angles-on-the-coordinate-plane.html
https://physics.info/vector-
components/practice.shtml
http://physics.bu.edu/~duffy/semester1/c3_vad
In the picture, we can say that the displacement is d_comp.html
the “short-cut” while the distance is the “long-cut”.
https://en.wikipedia.org/wiki/Displacement_(ge
That’s because displacement is a vector quantity
ometry)
and adding vectors is different from adding scalars.
When we add scalars, we simply add the numbers but in
vector addition, we consider
the direction.

30
SAN PEDRO RELOCATION CENTER NATIONAL HIGH SCHOOL – SCIENCE DEPARTMENT © 2020
WORKSHEET 3 – VECTOR ADDITION

I. Read and answer the following.

1. Air traffic controllers give instructions to airline pilots telling them in which direction
they are to fly. These instructions are called “vectors”. If these are the only instructions
given, is the name “vector” used correctly? Why or why not?
___________________________________________________________________________
___________________________________________________________________________
___________________________________________________________________________
2. Can the magnitude of a resultant vector be less than the magnitude of any of its
components? Explain.

___________________________________________________________________________
___________________________________________________________________________
___________________________________________________________________________

II. Solve the following problems. Show your complete solution on the box
provided.

1. A car is driven 225 km west and then 98 km southwest (45°). What is the displacement
of the car from the point of origin (resultant vector)? Draw a diagram.

2. A delivery truck travels 21 blocks north, 16 blocks east, and 26 blocks south. What is its
final displacement from the origin?

31
SAN PEDRO RELOCATION CENTER NATIONAL HIGH SCHOOL – SCIENCE DEPARTMENT © 2020
-
3. Vector A is 6.6 m long and points along the negative x axis. Vector B is 8.25 m long
and points at +55° to the positive x axis. What are the x and y components of each
vector?

4. Determine the Resultant Vector of the following vectors:


A = 26.5 m 56° NW; B = 44.0 m 28° NE; and C = 31.0 m S.

5. Hearing rattles from a snake, you make two rapid displacements of magnitude 1.8 m
and 2.4 m. Show how your two displacements might add up to give a resultant of
magnitude (a) 4.2 m; (b) 0.6 m; (c) 3.0 m.

REFERENCES:
Freedman, R.A. (2012). University Physics With Modern Physics. 13th Edition. SF California: Smith
Giancoli, D.C (2014). Physics, Principles with Applications. 7th Edition. USA. Smith

32
SAN PEDRO RELOCATION CENTER NATIONAL HIGH SCHOOL – SCIENCE DEPARTMENT © 2020
-
PHYSICS 1
Kinematics: Describing Motion

INSTANT TASK

Let’s recall your concepts on MOTION.


Read each statement carefully and fill in your
answers on the box. Complete answering the
questions and you will be able to find the
Mystery Word!

1. It is the STANDARD UNIT for DISTANCE


2. He was the PROPONENT of the LAWS of MOTION
3. It is defined as HOW FAST and in what DIRECTION an object is moving
4. another UNIT used for measuring TIME
5. It is defined as HOW FAR an object is moving:
6. It is defined as the RATE OF CHANGE OF VELOCITY
7. It is defined as the ACTION of CHANGING LOCATION or POSITION
8. It is defined as HOW FAR and in what DIRECTION an object is moving:
9. It is defined as HOW FAST an object is moving:

MYSTERY WORD:

SAN PEDRO RELOCATION CENTER NATIONAL HIGH SCHOOL – SCIENCE DEPARTMENT © 2020 33
DIRECT TALK

If you get the mystery word correctly, that would be MECHANICS. Mechanics is a
branch of Physics that studies about the relationships among force, matter and motion.
In this module we will study KINEMATICS, the part of mechanics that enables us to
describe motion. In the next modules we will study DYNAMICS, which relates motion to
its causes. For now, we will focus on the simplest kind of motion, a body moving along a
straight line (one-dimensional), and objects that move without rotating. Such motion is called
TRANSLATIONAL MOTION.

Any measurement of position, distance, or speed must be made with A body in


respect to a reference frame or frame of reference. This serves as the Physics is another word
basis for the measurement of position, distance or speed. We get different someone can use to
describe an object with
observations from different reference frames. (see Quick Connect) mass.

You learned from your JHS that there are 3 ways to describe motion in a straight line:
Speed/Velocity, Distance/Displacement, and Acceleration. Speed and distance are scalar quantities
while velocity, displacement and acceleration are vector quantities. We already defined that vector
quantities are quantities having both magnitude and direction, and the direction of an object can
be based on the Cartesian coordinate system (see PL: Vector Addition) In this module we will
discuss the equations to be used for these quantities.

DISTANCE AND DISPLACEMENT


The
Distance is defined as the total path travelled by an object while displacement is displacement may
defined as the change in position of the object. That is, displacement is how far an not equal the
object is from the starting point. To see the distinction between total distance total distance
travelled and displacement, imagine a person walking 70 m east and then turning travelled
around and walking back (west) a distance of 30 m. The total distance travelled is
100 m but the displacement is 40 m from the starting point. See Figure 7.1.

Consider the motion of an object over a particular time interval. Suppose


that at some initial time, call it t1, the object on the x axis at the position
x1 in the coordinate system as shown in Fig. 7.2. At some later time, t2,
suppose the object has moved to position x2,. The displacement of our
object is x2 – x1, and is represented by the arrow pointing to the right in
Fig. 7.2. It is convenient to write
x = x2 – x1, Figure 7.1 A person walking 70 m east
then 30 m, west. The total distance
where the symbol  (Greek letter delta) means “change in”. Then x travelled is 100 m (path shown dashed in
means “the change in x” or “change in position”, which is the red); but the displacement, shown as
displacement. The change in any quantity means the final value of that solid blue arrow, is 40 m to the east.

quantity, minus the initial value.

Now consider an object moving to the left as shown in Fig.7.3 Here the
object, starts at x1 = 30.0 m and walks to the left to the point x2 = 10.0
m. in this case, the displacement is
x = x2 –x1 = 10.0 m – 30.0 m = - 20.0 m
and the blue arrow representing the vector displacement points to the left.
For one-dimensional motion along the x axis, a vector pointing to the right Figure 7.2 The arrow represents the
displacement x2 – x1. Distances are in
is positive whereas a vector pointing to the left has negative sign. meters

34 SAN PEDRO RELOCATION CENTER NATIONAL HIGH SCHOOL – SCIENCE DEPARTMENT © 2020
AVERAGE VELOCITY

Another description of a moving object is how fast it is moving ―


its speed or velocity.

Speed refers to how far an object travels in a given time interval,


regardless of direction. If a car travels 240 km in 3 h, we say its
average speed was 80 km/h. In general, the average speed of an Figure 7.3 For the displacement
object is defined as the total distance travelled along its path x = x2 –x1 = 10.0 m – 30.0 m, the
divided by the time it takes to travel this distance: displacement vector points left.

𝐝𝐢𝐬𝐭𝐚𝐧𝐜𝐞 𝐭𝐫𝐚𝐯𝐞𝐥𝐥𝐞𝐝
𝐚𝐯𝐞𝐫𝐚𝐠𝐞 𝐬𝐩𝐞𝐞𝐝 =
𝐭𝐢𝐦𝐞 𝐞𝐥𝐚𝐩𝐬𝐞𝐝

In Physics, we make a distinction between speed and velocity. Speed is just a positive number,
with units while velocity is used to signify both the magnitude and direction, hence, a vector
quantity. Another difference is that the average velocity is defined in terms of displacement, rather
than the total distance travelled:

𝐝𝐢𝐬𝐩𝐥𝐚𝐜𝐞𝐦𝐞𝐧𝐭 𝐭𝐫𝐚𝐯𝐞𝐥𝐥𝐞𝐝 𝐟𝐢𝐧𝐚𝐥 𝐩𝐨𝐬𝐢𝐭𝐢𝐨𝐧 − 𝐢𝐧𝐢𝐭𝐢𝐚𝐥 𝐩𝐨𝐬𝐢𝐭𝐢𝐨𝐧 𝐱


𝐚𝐯𝐞𝐫𝐚𝐠𝐞 𝐯𝐞𝐥𝐨𝐜𝐢𝐭𝐲 (𝐯𝐚𝐯 ) = = =
𝐭𝐢𝐦𝐞 𝐞𝐥𝐚𝐩𝐬𝐞𝐝 𝐭𝐢𝐦𝐞 𝐞𝐥𝐚𝐩𝐬𝐞𝐝 𝐭

ACCELERATION

If the velocity of an object is changing, it is said to be accelerating. For instance, a car whose
velocity increases in magnitude from zero to 80 km/h is accelerating. Acceleration specifies how
rapidly the velocity of an object is changing. Since velocity is a vector quantity so acceleration is
also a vector quantity. Average acceleration is defined as the change in velocity divided by the time
taken to make this change:

𝐜𝐡𝐚𝐧𝐠𝐞 𝐢𝐧 𝐯𝐞𝐥𝐨𝐜𝐢𝐭𝐲 𝐟𝐢𝐧𝐚𝐥 𝐯𝐞𝐥𝐨𝐜𝐭𝐢𝐲 − 𝐢𝐧𝐢𝐭𝐢𝐚𝐥 𝐯𝐞𝐥𝐨𝐜𝐢𝐭𝐲 𝐯


𝐚𝐯𝐞𝐫𝐚𝐠𝐞 𝐚𝐜𝐜𝐞𝐥𝐞𝐫𝐚𝐭𝐢𝐨𝐧 (𝐚𝐚𝐯 ) = = =
𝐭𝐢𝐦𝐞 𝐞𝐥𝐚𝐩𝐬𝐞𝐝 𝐭𝐢𝐦𝐞 𝐞𝐥𝐚𝐩𝐬𝐞𝐝 𝐭

Example 7.1 Runner’s average velocity Example 7.2 Distance a cyclist travels
The position of a runner as a function of time is How far can a cyclist travel in 2.5 h along a straight
plotted as moving along the x axis of a coordinate road if her average velocity 18 km/h?
system. During a 3 s time interval, the runner’s
position changes from x1 = 50 m to x2 = 30.5 m. Solution:
What is the runner’s average velocity? Given:
Solution: t = 2.5 h
vav = 18 km/h
Identify first the given and the quantity that
In order to get the distance travelled, we need
we are looking.
to derive the formula by multiplying both sides
t = 3 s
with 𝐭:
x = x2 - x1 = 30.5 m – 50 m = -19.5 m 𝐱
vav = ? 𝐭 vav = 𝐭
−19.5 m 𝐭
vav = = −𝟔. 𝟓 𝐦/𝐬
3s 𝐱 = vav (t)
The displacement and average velocity are
negative, which tells us that the runner is 𝐱 = 18 km/h (2.5 h)
moving to the left along the x axis. The 𝐱 = 45 km
runner’s velocity is 6.5 m/s to the left.

SAN PEDRO RELOCATION CENTER NATIONAL HIGH SCHOOL – SCIENCE DEPARTMENT © 2020 35
Example 7.3 Car changes speed
A car travels at a constant 50 km/h for 100 km. It then speeds up to 100 km/h and is driven another
100 km. What is the car’s average speed for the 200-km trip?

Solution:

Given:
total distance 100 km + 100 km 200 km
vav = = =
time elapsed 2h+1h 3h
First travel:
v1 = 50 km/h
x1 = 100 km vav = 67 km/h
t1 = x1/v1 = 100 km/50 km/h = 2 h
Note: Averaging the two speeds by dividing it by 2 will
Second travel: give you the wrong answer. You must use the
v2 = 100 km/h definition for average speed.
x2 = 100 km
t2 = x2/v2 = 100 km/100 km/h = 1 h

Example 7.4 Average acceleration Example 7.5 Car slowing down


A car accelerates on straight road from rest to 75 An automobile is moving to the right along a
km/h in 5 s. What is the magnitude of its average straight highway, which we choose to be the
acceleration? positive x axis. Then the driver steps on the brakes.
If the initial velocity is vi = 15 m/s, and it takes 5 s
Solution: to slow down to vf =5 m/s, what was the car’s
average acceleration?
Given:
Solution:
t = 5 s
v = vf – vi = 75 km/h – 0 = 75 km/h Given:
* rest means the object is not moving v = 0 t = 5 s
v = vf – vi = 5 m/s – 15 m/s = -10 m/s
Since we have different units we need to
convert either the time or the velocity. Let’s v −10 m/s
convert the velocity. aav = = = -2 m/s2.
t 5s

v = 75 km/h = 21 m/s The negative sign appears because the final velocity
is less than the initial velocity. In this case the
v 21 m/s direction of the acceleration is to the left (negative
aav = = = 4.2 m/s2
t 5s x direction) even though the velocity is always
pointing to the right.

TRY THIS!
1. An ant starts at x = 20 cm on a piece of graphing paper Deceleration happens when
and walks along the x axis to x = -20cm. It then turns an object is slowing down. But
around and walks back to x = -10 cm. Determine (a) the be careful: deceleration does
not mean that the acceleration
ant’s displacement and (b) the ant’s distance traveled
is necessarily negative. We have
a deceleration whenever the
2. A car moves along the x axis. What is the sign of the magnitude of velocity is
car’s acceleration if it is moving in the positive x direction decreasing, thus velocity and
with (a) increasing speed or (b) decreasing speed? What acceleration point in opposite
is the sign of the car’s acceleration if it is moving in the directions when there is
deceleration.
negative x direction with (a) increasing speed or (b)
decreasing speed?

Check your answer: 1. (a) -30 cm (b) 50 cm; 2. (a) + (b) – (c) – (d) +

36
SAN PEDRO RELOCATION CENTER NATIONAL HIGH SCHOOL – SCIENCE DEPARTMENT © 2020
FLASH CHECK
Use the equations discussed above to solve these problems. Show your complete
solution.

1. A particle at t1 = - 2 s is at x1 = 4.8 cm and at t2 = 4.5 s at x2 = 8.5 cm.


What is the average velocity over this time interval?

2. A sports car accelerates from rest to 95 km/h in 4.3 s. What is its average
acceleration in m/s2?

QUICK
REFERENCES
CONNECT https://www.dreamstime.com/girl-student-sitting-
wooden-desk-chair-above-student-s-head-question-
mark-schoolgirl-looking-answer-to-image153473039

https://www.graphicsfactory.com/Clip-Art/Planner-
Stickers/reminder-digital-planner-sticker-
You read it right. Going back to the
409385.html
START UP, with respect to the
ground, you are not moving but https://www.quora.com/What-is-a-body-in-Physics-
Mechanics#:~:text=In%20physics%2C%20a%20bod
with respect to the Sun, you are
y%20is,force%20to%20change%20their%20velocity
moving with the Earth with a speed
of 2.97x104 m/s! Therefore we can say http://www.studyphysics.ca/newnotes/20/unit01_kin
that MOTION IS RELATIVE, that is, it ematicsdynamics/chp03_kinematics/lesson09.htm
depends on who is observing. That’s
https://www.pinterest.ph/pin/361836151311622239/
why frame of reference is of great
importance. Giancoli, D.C (2014). Physics, Principles with
Applications. 7th Edition. USA. Smith

SAN PEDRO RELOCATION CENTER NATIONAL HIGH SCHOOL – SCIENCE DEPARTMENT © 2020 37
PHYSICS 1
Kinematics: Uniform Accelerated Motion

Is there Physics
behind this
picture ― water
streaming down
the window?

INSTANT TASK

Examine these images:

IMAGE A IMAGE B

Describe the motion of the cars in the images in terms of their:

Image A Image B

VELOCITY

ACCELERATION

38 SAN PEDRO RELOCATION CENTER NATIONAL HIGH SCHOOL – SCIENCE DEPARTMENT © 2020
DIRECT TALK

Based from the images, we can assume that image A could either have constant
velocity, therefore zero acceleration or it can be increasing its speed at a constant rate
resulting to constant acceleration and Image B has changing velocity due to traffic
hence the acceleration is also changing. This leads us to two kinds of motion: Uniform and
Non-Uniform Motion.
Basis of
A body is said to be in uniform motion if Uniform Motion Non-Uniform Motion
difference
it travels equal distances in equal
When a body moves When a body moves
intervals of time, no matter how small
along a straight line
along a straight line
these time intervals may be. For example, Definition or with uniform speed but with variable or
a car is running at a constant speed say meaning or steady speed is change in speed is
20 m/s, will cover equal distances of 20 called Uniform called non-uniform
meters, every second, so its motion will be motion motion
uniform. On the other hand, a body is How much Covers equal distance
Covers unequal
said to be in a non-uniform motion if it distance is in equal intervals of
distances in equal
travels unequal distances in equal covered? time intervals of time
intervals of time. What is average Same as the actual Different from actual
speed here? speed of the object speed of the object
Under the Non-Uniform Motion we will Rectilinear Acceleration is non-
Acceleration is zero
encounter the terms Accelerated motion zero
Motion and Uniform Accelerated Table 8.1 Uniform Motion vs Non-Uniform Motion
Motion. In this module we will study
about Uniform Accelerated Motion.

Uniform Accelerated Motion Rectilinear motion


is motion in a straight
line.
A body has uniform acceleration if the velocity is changing at a constant rate.
The table below shows data for uniform and non-uniform acceleration.

Time (s) Velocity (m/s) Time (s) Velocity (m/s) Three ways to
0.0 0.0 0.0 0.0 accelerate: speed up,
1.0 2.0 1.0 2.0 slow down, and turn.
2.0 4.0 2.0 8.0
3.0 6.0 3.0 12.0
4.0 8.0 4.0 22.0
5.0 10.0 5.0 26.0 Uniform means
Constant
(a) (b)
Table 8.2 (a) Uniform accelerated motion (b) Non-uniform accelerated motion
When we are talking about motion in a straight line with uniform acceleration, there are five equations
of motion, which are helpful in solving unknown quantities. Take note that these kinematic equations are
only valid for uniform acceleration.
Where:
v = v0 + at v= average velocity for constant acceleration
1
x = x0 + v0t +2at2 v = final velocity
v0 = initial velocity
v2 = v02 + 2a(x – x0)
v+v t = elapsed time since initial time t0 is always zero
v= 2 0
x0 = initial position
v + v0
x = ( )t x = final position
2
*we used different notations from the ones we a = constant acceleration
used in PL-Kinematics: Describing motion.
x = (x – x0)
SAN PEDRO RELOCATION CENTER NATIONAL HIGH SCHOOL – SCIENCE DEPARTMENT © 2020 39
Example 2.1 Runway design
You are designing an airport for small planes. One kind of airplane that might use this airfield
must reach a speed before takeoff of at least 27.8 m/s, and can accelerate at 2.00 m/s 2. (a) if the
runway is 150 m long, can this airplane reach the required speed for takeoff? (b) If not, what
minimum length must the runway have?

Solution:
Identify first the given in the problem, the RTF (Required To Find), the equation to be used and
then solve.

Given: minimum v = 27.8 m/s; v0 = 0; a = 2.00 m/s2; x = 150 m


RTF: (a) required v (b) minimum x
Equations to be used for (a) and (b): v2 = v02 + 2a(x – x0)

(a)
v2 = v02 + 2a(x – x0)
v2 = (0)2 + 2(2.00 m/s2)(150 m) substitute the values and perform the operation
v2 = (4 m/s2)(150 m)
v2 = 600 m2/s2
v = 600 m2/s2
v = 24.5 m/s final answer

The runway length is not sufficient because the minimum speed for takeoff, 27.8 m/s, is required.

(b) Let’s derive the formula:

v2 = v02 + 2a(x – x0)


v2 - v02 = 2a(x – x0) transpose v02 to the other side of the equation

v2 - v0 2 2a (x - x0 )
= divide both sides by 2a
2a 2a

v2 - v0 2
2a
= (x – x0) final equation

m2
(27.8 s ) - (0)2
2(2.00 m/s2 )
= (x – x0) substitute the values and perform the operation

(x – x0) = 193 m final answer

A 200-m runway is more appropriate for this plane.

40
SAN PEDRO RELOCATION CENTER NATIONAL HIGH SCHOOL – SCIENCE DEPARTMENT © 2020
Example 2.2 Acceleration of a car
How long does it take a car to cross a 30.0-m wide intersection after the light turns green, if the
car accelerates from rest at a constant 2.00 m/s2

Solution:

Given:
x = 30.0 m; v0 = 0; a = 2.00 m/s2
RTF: t
1
Equation to be used: x = x0 + v0t +2at2

1
x = x0 + v0t +2at2 let’s assume x0 and v0 = 0

1
x = 2at2 cross multiply 2 with x and divide both sides by a

2x 2
=t substitute the values and perform the operation
a
2(30.0 m)
= t2
2.00 m/s2

30 s2 = t
t = 5.48 s final answer

Note: When we took the square root, we should have written  5.48 s but – 5.48 s is a time before our
chosen time interval (initial time = 0) and it makes no sense physically so we just ignore it.

TRY THIS!
Read and solve the following problems.

1. The Lady is 8 meters from the bus stop, when the Bus, starting from rest at the bus stop,
starts to move off with an acceleration of 0.4 m/s 2. What is the least speed at which the
Lady must run in order to catch the Bus?

2. An airplane accelerates down a runway at 3.20 m/s2 for 32.8 s until is finally lifts off the
ground. Determine the distance travelled before takeoff.

3. A car starts from rest and accelerates uniformly over a time of 5.21 seconds for a distance
of 110 m. Determine the acceleration of the car.

4. A race car accelerates uniformly from 18.5 m/s to 46.1 m/s in 2.47 seconds. Determine the
acceleration of the car and the distance travelled.

5. Rocket-powered sleds are used to test the human response to acceleration. If a rocket-
powered sled is accelerated to a speed of 444 m/s in 1.83 seconds, then what is the
acceleration and what is the distance that the sled travels?

Check your answer: 1) 2.53 m/s; 2) 1721. 34 m; 3) 8.10 m/s2; 4) 11.17 m/s2, 79.78 m; 5) 242.62 m/s2, 406.26 m

SAN PEDRO RELOCATION CENTER NATIONAL HIGH SCHOOL – SCIENCE DEPARTMENT © 2020 41
FLASH CHECK
A. Identify which among B. Solve the following problems. Show your complete
these situations is solution on a separate sheet of paper and write
having uniform/non- your final answer on the box provided.
uniform motion. Write 1. A bike accelerates uniformly from rest to a speed
UM for uniform motion of 7.10 m/s over a distance of 35.4 m. Determine
and NUM for non- the acceleration of the bike.
uniform motion
1. A horse running
_______
2. A falling apple from a
tree _______
3. A vibrating spring in a 2. A bullet leaves a rifle with a muzzle velocity of 521
sewing machine m/s. While accelerating through the barrel of the
_______ rifle, the bullet moves a distance of 0.840 m.
4. Earth moving around Determine the acceleration of the bullet (assume a
the Sun _______ uniform acceleration).
5. A train coming to its
terminal point
_______

QUICK REFERENCES
CONNECT https://www.driverseducationusa.com/resources/positionin
g-your-car-on-the-road/
https://daily.jstor.org/the-science-of-traffic/
https://www.kissclipart.com/stick-figure-thinking-clipart-
stick-figure-drawing-rj2z0r/download-clipart.html
https://www.jagranjosh.com/general-knowledge/what-is-
the-difference-between-uniform-and-nonuniform-motion-
1514455582-1
https://www.graphicsfactory.com/Clip-Art/Planner-
The motion of the droplets Stickers/reminder-digital-planner-sticker-409385.html
of water is an example of https://phys.libretexts.org/Bookshelves/Classical_Mechani
non-uniform motion! They cs/Book%3A_Classical_Mechanics_(Tatum)/06%3A_Moti
travel with unequal on_in_a_Resisting_Medium/6.02%3A_Uniformly_Accelera
distances in equal intervals ted_Motion
of time as it stream down
the window. https://www.physicsclassroom.com/class/1DKin/Lesson-
6/Sample-Problems-and-Solutions
Amazing!
https://www.pinterest.ph/pin/92534967330925307/

Giancoli, D.C (2014). Physics, Principles with Applications.


7th Edition. USA. Smith

42
SAN PEDRO RELOCATION CENTER NATIONAL HIGH SCHOOL – SCIENCE DEPARTMENT © 2020
PHYSICS 1
Free Fall

Why do
Skydivers use
parachutes??

INSTANT TASK

Which do you think will fall faster?

In this activity you will need a notebook

Tear one piece of paper from that same notebook. Then drop the
notebook and the piece of paper from the same height at the same time. Which reach the
ground first? __________________. Why do you think so? __________________________
__________________________________________________________________________
Of course! It’s the notebook because it’s more massive. Now try this! Crumple that same
paper and then drop them again, the notebook and the crumpled paper, from the same
height at the same time.
Notebook
What happened? Why is that? Write your observations below. is more
massive!

SAN PEDRO RELOCATION CENTER NATIONAL HIGH SCHOOL – SCIENCE DEPARTMENT © 2020 43
DIRECT TALK

So, the Notebook and the crumpled paper fell at the same time from the same height
even though they remained different in their masses. Why is that? That’s Right! Free Fall.
You already learned Free fall from your previous years, it’s when an object falls free from
any resistance ― no friction, with the air or otherwise ― and falls under the influence of gravity
alone. what we’re going to study in this module is how do we quantify the quantities involved in
Freely falling objects such as the time for an object to fall, its initial and final velocity, how far did the
object fall, and its acceleration. Let’s Begin!

Freely falling objects are under the Non-Uniform Motion with uniform
acceleration. Do you remember the five kinematic equations we discussed in
PL 8? We will also use these equations in free fall since the acceleration here
Gravity is the
is constant. The difference, though, is that the acceleration in free fall is due force by which the
to the influence of gravity. That’s why we call it acceleration due to Earth draws objects
gravity, denoted as g. It has a constant value of 9.8 m/s2, at surface of the toward its center.
Earth. Acceleration due to gravity changes as you move farther from the
center of the Earth. That’s why when you go to the moon, you tend to
“float”.

Below are the equations that we will use for freely falling objects. since the
motion is vertical, we will change the x with y.
Where:
v = v0 + gt v= average velocity for constant acceleration
1
y = y0 + v0t +2gt2 v = final velocity
v2 = v02 + 2g(y – y0) v0 = initial velocity
v+v
v= 2 0 t = elapsed time since initial time t0 is always zero
v + v0 y0 = initial position
y = ( )t y = final position
2
*we used different notations from the ones we used in PL- g = acceleration due to gravity, 9.8 m/s2
Kinematics: Describing motion.
y = (y – y0)

When choosing an equation to be used, make sure


that there is only one missing quantity to be sought.
You can choose any equation that is suitable for the
problem.

Acceleration due to gravity is a vector quantity, and


its direction is always pointing downward. Our
convention for signs will be different when dealing
with freely falling objects. See figure 9.1

Figure 9.1 Directions to be used for


acceleration due to gravity of freely falling
objects. If it’s moving downward, it’s positive
and if it is moving upward, it’s negative.

44
SAN PEDRO RELOCATION CENTER NATIONAL HIGH SCHOOL – SCIENCE DEPARTMENT © 2020
Example 9.1 Falling from a tower
Suppose that a ball is dropped (v0 = 0) from a tower. How far will it have fallen after a time
t1 = 1.00 s, t2 = 2.00 s, and t3 = 3.00 s? Ignore air resistance
Solution
Given: g = +9.8 m/s2 ; v0 = 0; y0 = 0 (initial position); t1 = 1.00 s; t2 = 2.00 s; t3 = 3.00 s
RTF: y1; y2; and y3.
1 2
Equation to be used: y = y0 + v0t + gt
2
1
y = y0 + v0t +2gt2 cancel out y0 and v0t since they’re equal to zero
1
y1 = 2gt2 substitute the values and perform the operation
1
y1 = 2(+9.8 m/s2) (1.00 s)2 = 4.9 m [t1 = 1.00 s]
1
y2 = 2(+9.8 m/s2) (2.00 s)2= 19.6 m [t2 = 2.00 s]
1
y3 = 2(+9.8 m/s2) (3.00 s)2= 44.10 m [t3 = 3.00 s]

Note: Whenever we say “dropped” it means v0 = 0.

Example 9.2 Thrown down from a tower


Suppose the ball in Example 5.1 is thrown downward with an initial velocity of 3.00 m/s, instead
of being dropped. (a) What then would be its position after 1.00 s and 2.00 s? (b) What would
its speed be after 1.00 s and 2.00 s? Compare with the speeds of a dropped ball.

Solution
Given: g = +9.8 m/s2 ; v0 = 3.00 m/s; y0 = 0 (initial position); t1 = 1.00 s; t2 = 2.00 s
RTF: (a) y1; y2 (b) v1; v2 (c) v1; v2 (dropped ball)
1 2
Equation to be used: y = y0 + v0t + gt
2
v = v0 + gt

(a)
1
y = y0 + v0t +2gt2 cancel out y0 since it’s equal to zero

1
y1 = (3.00 m/s)(1.00 s) +2(+9.8 m/s2) (1.00 s)2 = 7.9 m [t1 = 1.00 s]
1
y2 = (3.00 m/s)(2.00 s) +2(+9.8 m/s2) (2.00 s)2= 25.6 m [t2 = 2.00 s]

SAN PEDRO RELOCATION CENTER NATIONAL HIGH SCHOOL – SCIENCE DEPARTMENT © 2020 45
(b)
v = v0 + gt substitute the values and perform the operation
v1 = (3.00 m/s) + (+9.8 m/s )(1.00 s) = 12.8 m/s
2
[t1 = 1.00 s]
v2 = (3.00 m/s) + (+9.8 m/s )(2.00 s) = 22.6 m/s
2
[t2 = 2.00 s]

(c) Ball is dropped

v = v0 + gt substitute the values and perform the operation


v1 = (0 m/s) + (+9.8 m/s2)(1.00 s) = 9.8 m/s [t1 = 1.00 s]
v2 = (0.00 m/s) + (+9.8 m/s2)(2.00 s) = 19.6 m/s [t2 = 2.00 s]

Note: For both examples, the speed increases linearly in time by 9.8 m/s during each second. But
the speed of the downwardly thrown ball at any instant is always 3.00 m/s (its initial speed) higher
than that of a dropped ball.

Example 9.3 Ball thrown upward


A person throws a ball upward into the air with an initial velocity of 15.0 m/s. Calculate how
high it goes. Ignore air resistance.

Solution

Given: g = - 9.8 m/s2 (upward direction) ; v0 = 15.0 m/s; y0 = 0 (initial position); v= 0


RTF: y
Equation to be used: v2 = v02 + 2g(y – y0)

v2 = v02 + 2g(y – y0) derive the formula to get (y – y0)


v2 - v0 2
(y – y0) = substitute the values and perform the operation
2g
(0.00 m/s)
2 - (15.0 m/s)2
(y – 0)= = 11.5 m
2(-9.8 m/s2)

The ball reaches a height of 11.5 m above the hand.

For objects thrown upward, these are the things that you have to
consider.
 The acceleration due to gravity is always negative because of the
direction.
 As the ball rises, its speed decreases until it reaches the highest
point, where its speed is zero for an instant; then it descends, with t t2
increasing speed.
 Even if the speed at the highest point is zero, its acceleration g Figure 9.2 Direction of velocity
and acceleration for a ball
remains the same.
thrown up in the air.
 The time t is only the time it takes for the ball to reach the highest Acceleration from gravity is
always constant and
point, as it goes down, it travels for another time t2. So we have a downward, but the direction
total time T that the ball is in the air: T = t + t2
and magnitude of velocity
change.

46
SAN PEDRO RELOCATION CENTER NATIONAL HIGH SCHOOL – SCIENCE DEPARTMENT © 2020
Example 9.4 Ball thrown upward (II)
In Example 5.3, how long is the ball in the air before it comes back to the hand?

Solution

Given: g = - 9.8 m/s2 (upward direction) ; v0 = 15.0 m/s; y0 = 0 (initial position); v= 0


RTF: T (total time the ball is in the air)
Equation to be used: v = v0 + gt

Using this equation we will get t which is the time it takes to reach the highest point. Since in the
problem it reached that same position as the initial position, we can just multiply the time by 2.

v = v0 + gt substitute the values and perform the operation

−v 0
t=
g

t = −(15.0 m/s)
(−9.8 m/s2 )
= 1.53 s this is the time it takes for the ball to reach the highest point.

T = 2t = (2)(1.53 s) = 3.06 s you can use different equation and you’ll get the same results.

TRY THIS!
Read and solve the following problems.

1. Luke drops a pile of roof shingles from the top of a roof located 8.52 meters above the
ground. Determine the time required for the shingles to reach the ground.
Solve here:

2. Rex throws his mother's crystal vase vertically upwards with an initial velocity of 26.2 m/s.
Determine the height to which the vase will rise above its initial height.
Solve here:

Check your answer: 1) 1.32 s; 2) 35 m

SAN PEDRO RELOCATION CENTER NATIONAL HIGH SCHOOL – SCIENCE DEPARTMENT © 2020 47
FLASH CHECK
Read and answer the following. Write your answers on the space provided.

1. As a freely falling object speeds up, what is happening to its acceleration ― does it
increase, decrease, or stay the same? (a) Ignore air resistance (b) Consider air resistance
(a)___________________ (b) ___________________
2. Can an object have zero velocity and nonzero acceleration at the same time? Give
examples. _______________________________________________________________
3. A ball is dropped from the top of a tall building. At the same instant, a second ball is thrown
upward from the ground level. When the two balls pass one another, one on the way up,
the other on the way down, compare the magnitudes of their acceleration: (encircle your answer)
(a) The acceleration of the dropped ball is greater
(b) The acceleration of the ball thrown upward is greater
(c) The acceleration is the same
(d) The acceleration changes during the motion, so you cannot predict the exact value when
the two balls pass each other
(e) The acceleration are in opposite directions
4. A ball player catches a ball 3.4 s after throwing it vertically upward.
(a) With what speed did he throw it, and (b) what height did it reach?

QUICK REFERENCES

CONNECT
Hewitt, P.G. (2006). Conceptual Physics. 10th Edition. USA.
Pearson Education Inc
Giancoli, D.C (2014). Physics, Principles with Applications. 7th
Edition. USA. Smith
https://www.kindpng.com/imgv/bwToxT_parachuting-transparent-
cartoons-clipart-transparent-png-parachute-png/#gal_parachuting-
transparent-cartoons-clipart-transparent-png-parachute-png-
download_bwToxT_700476.png
https://www.dreamstime.com/vector-cartoon-stick-figure-drawing-
conceptual-illustration-falling-man-businessman-two-men-holding-
After about 12 seconds, you reach a speed where the force of safety-net-to-catch-image161481920
air resistance (pushing you upward) increases so much that it https://www.kissclipart.com/science-notebook-clip-art-clipart-
balances the force of gravity (pulling you downward). At that science-clip-art-a2dd39/
point, there is no net acceleration and you keep on falling at a https://www.pngwing.com/en/free-png-btijc
steady speed called your terminal velocity. https://spaceplace.nasa.gov/what-is-gravity/en/
https://www.graphicsfactory.com/Clip-Art/Planner-
Stickers/reminder-digital-planner-sticker-409385.html
https://www.khanacademy.org/science/ap-physics-1/ap-one-
dimensional-motion/falling-objects-ap-physics/a/freefall-ap1
https://www.physicsclassroom.com/class/1DKin/Lesson-
6/Kinematic-Equations-and-Free-Fall
https://study.com/learn/free-fall-questions-and-answers.html
http://www.problemsphysics.com/mechanics/motion/unif_acce_m
otion_solution.html#Solution_to_Problem_4
https://www.explainthatstuff.com/how-parachutes-work.html

48 SAN PEDRO RELOCATION CENTER NATIONAL HIGH SCHOOL – SCIENCE DEPARTMENT © 2020
PHYSICS 1
Graphing Motion

SPOT GOAL

 Discuss the different components of a graph


 Interpret velocity and acceleration, respectively, as
slopes of position vs. time and velocity vs. time curves
 Interpret displacement and velocity, respectively, as
areas under velocity vs. time and acceleration vs. time
curves

Philippines is located in the Pacific


Ring of Fire that’s why it is
susceptible to earthquakes. It’s
important to study Earthquake to
know more about their causes
and predict where they are likely
to happen in order for our
engineers to build safer buildings.
But how do Scientists study
Earthquakes?

SAN PEDRO RELOCATION CENTER NATIONAL HIGH SCHOOL – SCIENCE DEPARTMENT © 2020 49
INSTANT TASK

Plot the following points and answer the questions that follow.

Graph A

Data A
y - axis x- axis
1 2
2 4
3 6
4 8
5 10
6 12

Graph B

Data B
y - axis x- axis
3 2
3 4
3 6
3 8
3 10
3 12

50 SAN PEDRO RELOCATION CENTER NATIONAL HIGH SCHOOL – SCIENCE DEPARTMENT © 2020
Graph C

Data B

y - axis x- axis
1 2
2 5
3 6
4 9
5 13
6 14

Fill in the table with your answers.

1. Describe the data on the y –axis and the x - axis


2. Describe the graph
3. Identify the slope of the graph
rise y
 slope = =
run x

Graph A Graph B Graph C

Describe the data on y-axis

Describe the data on x-axis

Describe the graph

Identify the slope

SAN PEDRO RELOCATION CENTER NATIONAL HIGH SCHOOL – SCIENCE DEPARTMENT © 2020
51
DIRECT TALK

Drawing graphs is a very useful means of presenting information - and making it easily
understood. Changes and patterns can be quickly recognized. In physics, we often use
these graphs to represent objects in motion. These graphs are called motion graphs.
The three most common types of motion graphs are acceleration vs. time graphs, velocity vs.
time graphs and displacement vs. time graphs.
The graphs that you did in the INSTANT TASK ― straight diagonal line, straight horizontal
line and curved line ― are the usual types of graphs that we encounter in graphing motion. Based
on your answers, the slope for a straight diagonal line is always CONSTANT, for a straight
horizontal line, it’s always ZERO, and for curved lines, we can say that it’s CHANGING. In this
module we will describe the motion of an object graphically and identify what quantity slopes
represent in describing motion graphically.
VELOCITY vs TIME GRAPH
This graph shows how the speed of a moving object changes with time. You Slope describes
can't immediately determine where the object is from this graph. You can say the direction and
what direction it's moving, how fast it's going, and whether or not it's steepness of the line.
accelerating, however. The y-axis describes the velocity and the slope
The steeper the line,
describes the acceleration. The origin describes the motion of an object as
zero velocity meaning the object is at rest or not moving. If the graph is in the the higher the slope
first quadrant (+ y-axis) the object is moving forward. If the graph is in the
fourth quadrant (- y-axis), the object is moving in the opposite direction.
Let us describe the motion, in terms of velocity and acceleration, of the object
in the following graphs.
If the graph is here, the object is moving forward
The object is now moving forward. Speed
Speed is
is increasing, acceleration is changing
constant,
acceleration
is zero

Speed is
Speed is decreasing,
increasing, The object stops
acceleration
acceleration at an instant
is constant
is constant
Velocity (m/s)

0 The object reversed its x - axis


The object stops direction. Speed is Time (s)
at an instant increasing, acceleration is
constant Speed is
decreasing,
acceleration
Speed is is changing
constant,
If the graph is here, the object is moving backward acceleration
is zero
y - axis

Figure 10.1 Describing motion (velocity and acceleration) in velocity-time graph

52
SAN PEDRO RELOCATION CENTER NATIONAL HIGH SCHOOL – SCIENCE DEPARTMENT © 2020
POSITION vs TIME GRAPH
This graph shows how far an object has moved with time. The y-axis describes the displacement
and the slope describes the velocity. Any point from the y-axis describes the starting point of the
object. If the graph is in the first quadrant (+ y-axis) the object is moving forward. If the graph is in
the fourth quadrant (- y-axis), the object is moving in the opposite direction.

Object is moving forward speed


Object is on the same and acceleration is changing.
position, meaning it’s not
moving.

Object is Object is
moving moving
Position (m)

forward, speed backward,


is constant Speed is
acceleration is constant
0 zero acceleration is
zero x - axis
Time (s)

The object is at
its starting point Object is on the same
position, meaning it’s
y - axis
not moving.
Figure 10.2 Describing motion(position, velocity and acceleration) in position-time graph

ACCELERATION vs TIME GRAPH


This graph shows how an object accelerates with time. The y-axis describes the acceleration and its
slope doesn’t represent any quantity. The origin describes the motion of an object as zero
acceleration meaning the object moving towards the x-axis is slowing down or decelerating. If the
graph is in the first quadrant (+ y-axis) the object is moving forward. If the graph is in the fourth
quadrant (- y-axis), the object is moving in the opposite direction.
If the graph is here, the object is moving forward

Acceleration is
constant

Acceleration is Acceleration is
decreasing decreasing but in
Acceleration is
Acceleration (m/s2)

increasing opposite direction

0 Acceleration is x - axis
increasing but in
Time (s)
opposite direction
Acceleration is
zero
Acceleration is
constant

If the graph is here, the object is moving backward

y - axis

Figure 10.3 Describing motion (position, velocity and acceleration) in acceleration-time graph

SAN PEDRO RELOCATION CENTER NATIONAL HIGH SCHOOL – SCIENCE DEPARTMENT © 2020 53
AREA UNDER MOTION GRAPHS
As we have discussed, a velocity - time graph can be used to determine the acceleration of
an object by identifying its slope. Now, we will discuss how this velocity - time graph can also be
used to determine the displacement of an object. For velocity - time graphs, the area bound by the
line and the axes represents the displacement. We can also use the area under an acceleration -
time graph to determine its change in velocity. Consider Figure 10.4, the shaded regions are
the areas under the graph. In order to determine the
displacement, use the formulas for getting the area of the
respective shape. Use the same method when identifying the
velocity under acceleration - time graphs Area = b • h

Velocity (m/s)
Example 10.1 Calculating the Area of a Rectangle
Identify the displacement covered in the velocity-time graph below.

Given: Time (s)


(a)
velocity = 15 m/s (height)
25
t = 8 s (base)
Area = ½ • b • h
RTF: displacement
Velocity (m/s)

20
Solution: In order to get the

Velocity (m/s)
15 displacement, we use the formula for
10
area of a rectangle (Area = b • h)

5
Area = b • h = (8 s) • (15 m/s)
Time (s)
Area = 120 m (b)
2 4 6 8 10
Area = ½ • b • (h1 + h2)
Time (s) The displacement covered is 120 m.
-h2
Velocity (m/s)

Example 10.2 Calculating the Area of a Triangle


Identify the change in velocity in the acceleration-time graph below. -h1

Given:
acceleration= 12 m/s2 (height) Time (s)
(c)
t = 6 s (base)
15
RTF: change in velocity
Acceleration (m/s )
2

Figure 10. 4 Areas under motion


12 Solution: In order to get the change graphs. (a) Rectangle, (b) Triangle,
in velocity, we use the formula for (c) Trapezoid
9
area of a triangle (Area = ½ • b • h)
6
Area = ½ • b • h
3
Area = ½ • (6 s) • (12 m/s2)
Area = 36 m/s
2 4 6 8 10
Time (s) The change in velocity is 36 m/s. In geometry,
the area can be defined
36 m/s is just the change in velocity, what if we are to identify the final as the space occupied
velocity of the object? We can use the formula v = vf - vi. In this by a flat shape or the
example let’s assume that the initial velocity at t = 0 s is 20 m/s.
surface of an object.
Deriving the formula we get vf = v + vi

vf = v + vi = 36 m/s + 20 m/s = 66 m/s

The final velocity for the time interval 0 s – 6 s is 66 m/s.

54
SAN PEDRO RELOCATION CENTER NATIONAL HIGH SCHOOL – SCIENCE DEPARTMENT © 2020
Example 10.3 Calculating the Area of a Trapezoid
Identify the displacement from t = 4 s to t = 10 s in the velocity-time graph below.
Given:
20 velocity @ 4 s = 8 m/s (h1)
velocity @ 10 s = 20 m/s (h2)
16 t = 6 s (base)
RTF: displacement from t = 4 s to t = 10 s
Velocity (m/s)

12 Solution: In order to get the displacement from t = 4 s to t = 10 s,


we use the formula for area of a trapezoid (Area = ½ • b • (h1 + h2)
8
Area = ½ • b • (h1 + h2)
4 Area = ½ • (6 s) • (8 m/s + 20 m/s)
Area = 84 m
2 4 6 8 10
Time (s)
The displacement from t = 4 s to t = 10 s is 84 m.

Example 10.4 Calculating the Total Area


Identify the total change in velocity in the acceleration-time graph below.

20

16
Acceleration (m/s )
2

12

8
A B C
4

Time (s)
2 4 6 8 10 12 14 16

Given:
(A) acceleration @ 4 s = 12 m/s2 (height); t = 4 s (base)

(B) acceleration @ 4 s – 8 s = 12 m/s2 (height); t = 4 s (base)

(C) acceleration @ 8 s – 10 s = 12 m/s2 (height); t = 2 s (base)

RTF: Total change in velocity

Solution: In order to get the total change in velocity, need to identify first the area for A, B and C.

(B) Area = ½ • b • h (C) Area = b • h (A) Area = ½ • b • h


Area = ½ • (4 s) • (12 m/s2) Area = (4 s) • (12 m/s2) Area = ½ • (2 s) • (12 m/s2)
Area = 24 m/s Area = 48 m/s Area = 12 m/s

After getting the area of each section, compute the total change in velocity by getting the sum of the area of
each section.

Total change in velocity = 24 m/s + 48 m/s + 12 m/s = 84 m/s

SAN PEDRO RELOCATION CENTER NATIONAL HIGH SCHOOL – SCIENCE DEPARTMENT © 2020 55
FLASH CHECK
Study the graph below. Describe the acceleration and the
direction of the object on the
following time intervals:

Time
Acceleration Direction
interval
0-2 sec
2-4 sec
4-5 sec
5-9 sec
9-10 sec

Identify the displacement during


the time interval: 0 – 4 s

QUICK REFERENCES
CONNECT https://www.manula.com/manuals/fxhome/hitfilm-
pro/13/en/topic/grid
https://physics.info/motion-graphs/
The Study of Earthquake is known
https://www.graphicsfactory.com/Clip-Art/Planner-
as Seismology. When studying Stickers/reminder-digital-planner-sticker-409385.html
Earthquakes, seismologists https://www.ck12.org/c/physics/graphing-motion/lesson/Graphing-
uses seismic travel-time Motion-PPC/?referrer=concept_details
curves. A travel-time curve https://www.pinterest.ph/pin/740138519984231237/
is a graph of arrival times,
https://www.iris.edu/hq/inclass/video/travel_time_curves_describe
commonly P or S waves, recorded at different d
points as a function of distance from the seismic
https://www.ck12.org/c/earth-science/earths-core/rwa/Discovering-
source. Seismic velocities within the earth can be the-Core/
computed from the slopes of the resulting curves.
https://www.sciencelearn.org.nz/videos/332-why-do-scientists-
Because of these velocities, seismologists can use the study-earthquakes
direction and the difference in the arrival times
https://www.physicsclassroom.com/class/1DKin/Lesson-
between P-waves and S-waves to determine the 4/Determining-the-Area-on-a-v-t-Graph
distance to the source of an earthquake.
https://www.splashlearn.com/math-vocabulary/geometry/area
https://www.khanacademy.org/science/physics/one-dimensional-
motion/acceleration-tutorial/a/what-are-acceleration-vs-time-
graphs

56 SAN PEDRO RELOCATION CENTER NATIONAL HIGH SCHOOL – SCIENCE DEPARTMENT © 2020
WORKSHEET 4 – GRAPHING MOTION (SLOPE)

I. Study the following Motion Graphs and answer the questions correctly.

d (m) d (m) d (m)


40 40 40

30 30 30

20 20 20

10 10 10

0 t (s) 0 t (s) 0 t (s)


1 2 3 1 2 3 1 2 3

A B C
1. Which of the graphs shows that one of runners started 10 yards further ahead of the other? Explain
your answer.

2. In which of the following graphs below are both runners moving at the same speed? Explain your
answer.

II. Match the descriptions with the graphs.

Descriptions:

1. The car stopped


2. The car is travelling at a constant speed
3. The speed of the car is decreasing
4. The car is coming back

d (m) d (m) d (m) d (m)

t (s) t (s) t (s) t (s)


A B C D

SAN PEDRO RELOCATION CENTER NATIONAL HIGH SCHOOL – SCIENCE DEPARTMENT © 2020 57
-
III. Construct a Velocity-time graph for the following descriptions of motion.

1. The car stopped


2. The car is travelling at a constant speed
3. The car is accelerating
4. The car is slowing down

IV. Study the graph below and answer the following questions.

1. Which runner won the race? Explain your answer.


2. Which runner stopped for a rest? Explain your answer.
3. How long was the stop? Explain your answer.
4. How long did Bob take to complete the race? Explain your answer.

REFERENCES:
https://camillasenior.homestead.com/motion_graphs.pdf

58
SAN PEDRO RELOCATION CENTER NATIONAL HIGH SCHOOL – SCIENCE DEPARTMENT © 2020
-
WORKSHEET 5 – GRAPHING MOTION (AREA)

Study the Graphs below and answer the questions that follow.

1. The velocity-time graph shown above was created by a toy train which starts out moving north.
What was the displacement of the train in 25 seconds?

2. The acceleration-time graph shown above was created by a 3.27 kg Raichu running horizontally
with an initial velocity of -10 m/s. What was the final velocity of Raichu at t = 6 s?

REFERENCES:
https://www.monroe.k12.nj.us/cms/lib/NJ01000268/Centricity/Domain/322/Motion%20Graphs%20Worksheet-4.pdf
https://www.dropbox.com/s/vrhhkwd84l2yg6g/avs.txvs.tvvs.tworksheets1Dmotionvariables.pdf?dl=0

SAN PEDRO RELOCATION CENTER NATIONAL HIGH SCHOOL – SCIENCE DEPARTMENT © 2020 59
-
PHYSICS 1
Projectile Motion

INSTANT TASK

Are you a Sports person? What Sports do you play? Are you into ball games? Well, whether you’re into
Sports or not, we can’t remove the fact that there is Physics in Sports. Look at the common Sports Balls
below and answer the statements that follow. Later on, we will discuss how Physics is present in these
Sports.

B
D

C
A

1. Identify the Sports to which they belong to.


2. Give at least 5 similarities with these Sports Balls when used in a game.

60
SAN PEDRO RELOCATION CENTER NATIONAL HIGH SCHOOL – SCIENCE DEPARTMENT © 2020
DIRECT TALK

In the previous lessons, we studied the one – dimensional motion of an object in


terms of displacement, velocity, and acceleration, including purely vertical motion of a
falling object undergoing acceleration due to gravity. Now, we will study the motion of
objects moving through the air in two – dimensions near the Earth’s surface, such as a baseball, a
basketball, a tennis ball and a golf ball. (By the way, these are the Sports Balls in the INSTANT
TASK). These are all examples of Projectile Motion, which we can describe as taking place in two
dimensions if there is no wind.

In our analysis of Projectile motion, we will not be concerned now Two – dimensional motion
with the process by which the object is thrown or projected. We is where an object undergoes
consider only its motion after it has been projected, and before motion along the x and y axes “at
it lands or is caught  that is, we analyze our projected object the same time”
only when it is moving freely through the air under the action of
gravity alone (air resistance is ignored). Then the acceleration of
the object is that due to gravity which acts downward with
magnitude g = 9.8 m/s2, and we assume it is constant. Projectile is any body that
is given an initial velocity and then
follows a path determined entirely
Let us look at a ball rolling off the end of a horizontal table with by the effects of gravitational
an initial velocity in the horizontal (x) direction, vx0. See Fig. 11. 1, acceleration and air resistance.
where an object falling vertically is shown for comparison. The
velocity vector v at each instant points in the direction of the ball’s
motion at that instant, and thus is tangent to the path. We treat
the horizontal and the vertical components of velocity and Trajectory is the parabolic
acceleration separately, and we can apply the kinematic equations path followed by a projectile.
to the x and y components of the motion.

First we examine the vertical (y) component of the motion. At the


instant the ball leaves the table’s top (t = 0), it has only an x
component of velocity. Once the ball leaves the table (t = 0), it
experiences a vertically downward acceleration g. Thus vy is
initially zero but increases continually in the downward direction
(until the ball hits the ground). Let us take y to be positive
downward. Then g is in the - y direction, so ay = +g. (This
convention is the same with our convention of g in free fall)

In the horizontal (x) direction, on the other hand, there is no


acceleration (we are ignoring air resistance). With ax = 0, the
horizontal component of velocity, vx, remains constant, equal to
its initial value, vx0, and thus has the same magnitude at each
Figure 11.1 Projectile motion of a
point on the path. small ball projected horizontally with
initial velocity vx = vx0. The dashed
One result of this analysis is that an object projected line represents the path of the object.
horizontally will reach the ground in the same time as an The velocity vector v is in the
object dropped vertically. This is because the vertical motions direction of the motion at each point,
and thus is tangent to the path. The
are the same in both cases as shown in Fig. 11.1 velocity vectors are solid arrows, and
the velocity components are dashed.
(A vertically falling object is shown at
the left for comparison; vy is the same
at each instant for the falling object
and the projectile.)

SAN PEDRO RELOCATION CENTER NATIONAL HIGH SCHOOL – SCIENCE DEPARTMENT © 2020 61
If an object is projected at an upward angle 0, as in Figure 11.2, the analysis is similar,
except that now there is an initial vertical component of velocity, vy0. Because of the downward
acceleration of gravity, the upward component of velocity vy gradually decreases with time until
the object reaches the highest point on its path, at which point vy = 0. Subsequently the object
moves downward and vy increases in the downward direction as shown. As before, vx remains
constant. Let us take y to be negative upward. Then g is in the +y direction, so ay = -g.

Figure 11.2 Path of a projectile


launched with initial velocity v0 at an
angle 0 to the horizontal. Path is
shown dashed in black, the velocity
vectors are solid arrows, and the
velocity components are dashed. The
figure does not show where the
projectile hits the ground (at that
point, projectile motion ceases)

These are the Kinematic Equations for Projectile Motion:

Horizontal Motion Vertical Motion Where:


(ax = 0, vx = constant) (ay = g = constant) vx = Horizontal final Velocity Note:
x = Horizontal final position x = x – x0
vx = vx0 vy = vy0 + gt
vy = Vertical final velocity y = y – y0
x = x0 + vx0t y = y0 + vy0t + ½ gt2 y = Vertical final position
vy2 = vy02 + 2g(y - y0) *the subscript 0 means “t = 0.”

If the projection angle 0 is chosen relative to the +x axis, then

vx0 = v0 cos 0, and vy0 = v0 sin 0

Example 11.1 Calculating the Area of a Rectangle

A movie stunt driver on a motorcycle speeds


horizontally off a 50.0 m high cliff. How fast must the
motorcycle leave the cliff top to land on level ground
below, 90.0 m from the base of the cliff where the
cameras are? Ignore air resistance.
Given:
y = 50.0 m (height) RTF:
x = 90.0 m vx = vx0
g = 9.8 m/s2
Figure 11.2 Example 11.1
Solution:
The equation that we will use for the initial velocity is x = x0 + vx0t. But before we use this, let’s
use this equation y = y0 + vy0t + ½ gt2 first to get the time t.
y = y0 + vy0t + ½ gt2 transpose y0 to the other side of the equation
y = vy0t + ½ gt2 cancel vy0t since vy0 = zero
y = ½ gt2 derive the formula to get t
2( y )
t = substitute the values
g

62
SAN PEDRO RELOCATION CENTER NATIONAL HIGH SCHOOL – SCIENCE DEPARTMENT © 2020
2(50 .0 m )
t = m2
= 3.19 s remember our g is positive since our y is downward
(9.8 )
s

Knowing the time t, we can now use x = x0 + vx0t to get vx0

x = x0 + vx0t transpose x0 to the other side of the equation


x = vx0t derive the formula to get vx0
x
vx0 = substitute the values
t
(90.0 m)
vx0 = = 28.2 m/s
3.19 s

The motorcycle must have an initial velocity of 28.2 m/s in order to reach to ground 90.0 m from
the base.
Example 11.2 A kicked football

A kicked football leaves the ground at an angle of 37° with a velocity of 20 m/s as shown in the
Fig. 11.3. Calculate (a) the maximum height, (b) the time of travel before the football hits the
ground, and (c) how far away it hits the ground. Assume the ball leaves the foot at ground
level, and ignore air resistance and rotation of the ball.

Figure 11.3 Example 11.2

Given: RTF:
0= 37° (a) max. height (b) total time T (c) x
v0 = 20 m/s
g = -9.8 m/s2

Solution:
Before we solve for the RTF, Let’s identify first the components of the initial velocity, v0.
vx0 = v0 cos 0 = (20 m/s) (cos 37°) = 15. 97 m/s
vy0 = v0 sin 0 = (20 m/s) (sin 37°) = 12.04 m/s

The equation that we will use for the maximum height is y = y0 + vy0t + ½ gt2 but we need to
find t at the max height first. Let’s use this equation vy = vy0 + gt

vy = vy0 + gt cancel vy since vy = 0 at the max height


-vy0 = gt transpose vy0 to the other side of the equation
−vy0
t= derive the formula to get t and substitute the values
g
m
−(12.04 s )
t= m2
= 1.23 s
−9.8 s

SAN PEDRO RELOCATION CENTER NATIONAL HIGH SCHOOL – SCIENCE DEPARTMENT © 2020 63
Let’s now get the max. height.

y = y0 + vy0t + ½ gt2 transpose y0 to the other side of the equation


y = vy0t + ½ gt2 substitute the values

y = (12.04 m/s)(1.23 s) + ½ (-9.8 m/s2)(1.23 s)2


y = (14.81 m) + (-7.41 m) = 7.39 m

For the total time T or the time the ball is in the air, we just multiply the time t at max.
height by 2.

T = 2t = 2 (1.23 s) = 2.46 s

For the horizontal displacement, we use the equation x = x0 + vx0t.

x = x0 + vx0t transpose x0 to the other side of the equation


x = vx0t substitute the values
x = (15.97 m/s)(2.46 s)
We use T = 2.46 s for the time since we’re talking about the displacement from x0 to x.

x = 39.29 m

This horizontal distance traveled by the football is called the horizontal range R. If y = y0,
we can use this formula for the range:

v0 2 sin 2 0
R=
│g│

Example 11.3 Range of a canon ball

Suppose one of Napoleon’s cannons had a muzzle speed, v0, of 60.0 m/s. At what angle should
it have been aimed (ignore air resistance) to strike a target 320m away?

Given: v0 = 60 m/s; g = -9.8 m/s2; R = 320 m RTF: 0


Solution:
v0 2 sin 2 0
Let’s derive this formula R = to get the angle.
│g│

v0 2 sin 2 0 │g│(R)
R= 2 0 = sin-l
│g│ v0 2
m2
│−9.8 │(320 m)
-l
│g│(R) = v0 sin 22
0 2 0 = sin s
m 2
(60 s )

│g│(R)
sin 2 0 = 2 0 = 60.59° Figure 11.4 There are generally
v0 2 two angles 0 that will give the
same range. If one angle is 01, the
│g│(R) other is 02 = 90° - 01
2 0 = sin-l 𝟎 = 30.29° or 59.71°
v0 2 See Fig 11.4

64
SAN PEDRO RELOCATION CENTER NATIONAL HIGH SCHOOL – SCIENCE DEPARTMENT © 2020
FLASH CHECK
Read and solve the following problems. Show your complete solution.

1. A tiger leaps horizontally from a 7.5 m-high rock with a speed of 3.0 m/s. How far
from the base of the rock will she land?

2. A ball thrown horizontally at 12.2 m/s from the roof of a building lands 21.0 m from
the base of the building. How high is the building?

3. A fire hose held near the ground shoots water at a speed of 6.5 m/s. At what
angle(s) should the nozzle point in order that the water land 2.5 m away? Why are
there two different angles?

QUICK REFERENCES
CONNECT https://www.vectorstock.com/royalty-free-
vector/sports-balls-vector-466800

https://en.wikibooks.org/wiki/Fundamentals
_of_Physics/Motion_in_Two_Dimensions#
:~:text=Two%20dimensional%20motion%2
0is%20where,(x%2Cy)%7D%20coordinate

https://en.wikipedia.org/wiki/Diwata-1

https://www.nasa.gov/audience/forstudent
DIWATA-1 s/5-8/features/nasa-knows/what-is-a-
The first Philippine microsatellite satellite-58.html
and the first satellite built and
designed by Filipinos.
Giancoli, D.C (2014). Physics, Principles
with Applications. 7th Edition. USA. Smith.

65
SAN PEDRO RELOCATION CENTER NATIONAL HIGH SCHOOL – SCIENCE DEPARTMENT © 2020
WORKSHEET 6 – PROJECTILE MOTION

I. Read and answer the following conceptual questions.

1. The velocity of a typical projectile can be represented by horizontal and vertical components.
Assuming negligible air resistance, the horizontal component along the path of the projectile

(a) Increases. (b) Decreases. (c) Remains the same. (d) Not enough information.

2. When no air resistance acts on a fast-moving baseball, its acceleration is

(a) Downward, g.
(b) A combination of constant horizontal motion and accelerated downward motion.
(c) Opposite to the force of gravity.
(d) Centripetal.
3. A bullet fired from a rifle begins to fall

(a) As soon as it leaves the barrel.


(b) After air friction reduces its speed.
(c) Not at all if air resistance is ignored.

4. A baseball player hits a ball that soars high into the air. After the ball has left the bat, and while
it is travelling upward (at point P in the figure below), what is the direction of acceleration?
Ignore air resistance.

5. A baseball is hit high and far. Which of the following statements is true? At the highest point,

(a) The magnitude of acceleration is zero.


(b) The magnitude of velocity is zero.
(c) The magnitude of the velocity is the slowest.

6. Which statements are not valid for a projectile? Take up (y) as negative.

(a) The projectile has the same x velocity at any given point on its path.
(b) The acceleration of the projectile is positive and decreasing when the projectile is moving
upwards, zero at the top, and increasingly negative as the projectile descends.
(c) The acceleration of the projectile is a constant negative value.
(d) The y component of the velocity of the projectile is zero at the highest point of the
projectile’s path.
(e) The velocity at the highest point is zero.

66
SAN PEDRO RELOCATION CENTER NATIONAL HIGH SCHOOL – SCIENCE DEPARTMENT © 2020
-
II. Solve the following problems. Show your complete solution.

1. A rock is thrown horizontally off a 100 m cliff. It lands 95 m away. At what speed was it
thrown?

2. Farmer Bob and his horse flash jump a 150 m wide canyon. The flight through the air takes
8 s. Find the initial velocity (that is, the magnitude and angle of the initial velocity)

3. A grasshopper hops along a level road. On each hop, the grasshopper launches itself at
angle 0 = 45° and achieves a range R = 0.80 m. What is the average horizontal speed of
the grasshopper as it hops along the road? Assume that the time spent on the ground
between hops is negligible.

4. A projectile is fired with initial speed of 36.6 m/s at an angle of 42.2° above the horizontal
on a long flat firing range. Determine (a) the maximum height reached by the projectile,
(b) the total time in the air, (c) the range, and (d) the speed of the projectile 1.50 s after
firing.

5. A rescue plane wants to drop supplies to isolated mountains climbers on a rocky ridge 235
m below. If the plane is travelling horizontally with a speed of 69.4 m/s, how far in
advance of the recipients (horizontal distance) must the goods be dropped?

III. With the knowledge of projectile motion in mind, observe the different
scenarios at your house and give 5 situations that you think is an example of
projectile motion and give a brief explanation as to why you considered it as
an example of projectile motion.

REFERENCES:
http://www.phys.ttu.edu/~ritlg/courses/p1401/Hewitt_12/chapters/10_LectureOutline.pdf
https://www.youtube.com/watch?v=CJYHrHQgQUQ
Giancoli, D.C (2014). Physics, Principles with Applications. 7th Edition. USA. Smith.

SAN PEDRO RELOCATION CENTER NATIONAL HIGH SCHOOL – SCIENCE DEPARTMENT © 2020 67
PHYSICS 1
Forces: Causes of Motion

INSTANT TASK

We all have knowledge on what Force is  any kind of push or pull on an object. Below are examples
of different instances in our daily lives. Identify all the forces acting on objects in each situation.

1) Plate on a Table 2) Hanging chandelier

3) People Standing 4) Flying leaves

68
SAN PEDRO RELOCATION CENTER NATIONAL HIGH SCHOOL – SCIENCE DEPARTMENT © 2020
DIRECT TALK

In the previous lessons we have discussed about Kinematics, the study on descriptions
of motion. In this lesson, we will study about Dynamics, the study on the causes of
motion. In the INSTANT TASK you were asked to identify the Forces present in a
system. How many forces were you able to identify? That’s right, we have more than one
force acting on a system. In order for us to identify all the forces acting on a system, we
need to know first the Types of Forces and how to draw a Free-Body Diagram.

A Force is an interaction between two bodies or a body and its


environment. That’s why we always refer to the force that one body
exerts on a second body. When you push a plate across the table, you A System is used in Physics
exert a force on the plate. Force is a vector quantity; you can push or as whatever portion of the universe
pull a body in different directions. you choose to analyze.

When a force involves direct contact between two bodies, such as


push or pull that you exert on an object with your hand, we call it a
contact force. There are three types of contact forces: Normal
Force, Friction Force, and Tension Force. Normal Force is exerted
on an object by any surface with which it is in contact. The
(a) adjective normal means that the force always acts perpendicular to
the surface of contact, no matter what the angle of that surface. By
contrast, the Friction force exerted on an object by a surface acts
parallel to the surface, in the direction that opposes sliding. Friction
exists between two solid surfaces because even the smoothest
looking surface is quite rough on a microscopic scale. There are
(b) generally 3 types of friction: Rolling Friction, for round objects
rolling on the floor; sliding or kinetic friction, for objects that are
moving/sliding; and static friction, a force parallel to the two
surfaces that can arise even when they are not sliding. Suppose
you are pushing a desk and it doesn’t move, that is because of the
static friction exerted by the floor on the desk. The pulling force
(c) exerted by a stretched rope or cord on an object to which it is
attached is called a Tension force.

In addition to contact forces, there are long-range forces that act


even when the bodies are separated by empty space. The force
between two magnets is an example as is the force of gravity. The
gravitational force that the earth exerts on your body is called your
weight. The SI unit for force is newton, abbreviated N.
(d)

Figure 12.1 Common types of forces. (a) Identifying the right force is important when solving problems
Normal Force, (b) Friction force, (c) involving forces. In order to do so, we need to draw a diagram
Tension force, and (d) Weight showing all the forces acting on each object involved. Such a
diagram is called a free-body diagram or force diagram.

When drawing a free-body diagram, you choose one object, and draw an arrow to represent each
force acting on it. Include every force acting on that object. Do not show forces that the chosen object
exert on other objects. To help you identify each and every force that is exerted on your chosen
object, ask yourself what other object could exert a force on it. If your problem involves more than
one object, a separate free-body diagram is needed for each object.

SAN PEDRO RELOCATION CENTER NATIONAL HIGH SCHOOL – SCIENCE DEPARTMENT © 2020 69
Let’s draw a free-body diagram of a plate on a FN
table. Remember that Forces are vector quantity
and is represented by an arrow. The length of the
arrows here are the same, meaning they have the Figure 12.2
same magnitudes but pointing in opposite Free-body
directions. Take note however that these are not diagram of a
action-reaction pairs because these forces are plate on a table
acting on the same object, the plate. In this
free body-body diagram we can see that the
normal force FN and the gravitational force FG are
all on the y-axis. If we exert an applied force to
the right, this will give us with additional forces:
the applied force FA and the friction force Ff. FG
We already know that forces causes motion or a
change in the state of motion, and we can prove FN
that mathematically by finding the net force in a
system. Net force is the vector sum of the
forces acting on an object.

Suppose the plate has a mass of 1 kg. The


applied force is 5 N and the friction force is 3N. Ff FA
Let’s identify if the plate will accelerate.

Let’s get first the net force along the y direction:


Figure 12.3
∑Fy = FN + (-FG) Free-body

To get the FG, we use the formula FG diagram of a


plate on a table
with applied
FG = mg =(1)(9.8m/s2) = 9.8 N force FA and
friction force Ff
∑Fy = FN + (-FG) = (9.8 N) + (-9.8 N) = 0

Since the FG and the FN are the same, the net force along the y direction is zero. Zero net force
means there is no acceleration along the y direction. This is why the plate is not moving up or
down. Let’s now solve for the net force along the x direction:

∑Fx = FA + (-Ff)
∑Fx = (5 N) + (-3 N)
∑Fx = 2 N
This means that the plate will accelerate to the positive x direction with a net force of 2 N.

Draw free-body diagram on the remaining objects in the INSTANT TASK.

70
SAN PEDRO RELOCATION CENTER NATIONAL HIGH SCHOOL – SCIENCE DEPARTMENT © 2020
FLASH CHECK
Read and answer the following.

1. A hockey puck is sliding at constant velocity across a flat horizontal ice


surface that is assumed to be frictionless. Which of the sketches below is the
correct free body-diagram for this puck? What would your answer be if the
puck slowed down? Explain your answers.

QUICK REFERENCES
CONNECT https://sciencetrek.org/sciencetrek/topics/force_and_motion/f
acts.cfm

Forces are literally EVERYWHERE! https://www.dreamstime.com/stock-image-empty-white-plate-


wooden-table-image27430301
From the vastness of the universe,
where we have the gravitational https://www.dimensions.com/collection/people-standing
forces of the enormous stars and
https://www.pngkey.com/download/u2w7u2t4u2q8q8t4_fall-
planets, to the tiniest subatomic leaves-flying-in-air-free-clip-art/
particles of our bodies. There is force
when we sit, stand, and walk. When https://www.vectorstock.com/royalty-free-vector/textured-
cartoon-doodle-of-a-wooden-chair-vector-24121660
we open the doors or the windows,
there is force. When we play our https://www.lampsplus.com/products/allier-28-inch-high-
favorite sports, there is force. Even as wood-iron-outdoor-hanging-chandelier-light__56n34.html
you read this, there is force. How Giancoli, D.C (2014). Physics, Principles with Applications.
amazing is that! 7th Edition. USA. Smith

Freedman, R.A. (2012). University Physics With Modern


Physics. 13th Edition. SF California: Smith

SAN PEDRO RELOCATION CENTER NATIONAL HIGH SCHOOL – SCIENCE DEPARTMENT © 2020 71
PHYSICS 1
Newton’s Laws of Motion

INSTANT TASK

Identify the Laws of Motion that is evident in the following situations and explain why.

1. Riding a jeepney.
2. Pushing a grocery cart.
3. Walking.

72
SAN PEDRO RELOCATION CENTER NATIONAL HIGH SCHOOL – SCIENCE DEPARTMENT © 2020
DIRECT TALK

Let’s have a little review on the three Laws of Motion.

Newton’s First Law of Motion: The Law of Inertia

Every object continues in its state of rest, or of uniform velocity in a


straight line as long as no net force acts on it.

This is the reason for seatbelts. If you don’t wear seatbelts, you tend to move forward if the car
abruptly stops. Why do you tend to move forward? That’s because you have inertia  the natural
tendency of an object to resist changes in its state of motion. The measure of inertia is mass,
meaning, the more mass, the more inertia the body has.

Newton’s First Law does not hold in every reference frame. It only holds on inertial reference
frame – a reference frame that is either at rest or moving with constant velocity. If the frame of
reference is accelerating (noninertial reference frame), the Law of inertia is no longer valid.

Newton’s Second Law of Motion: The Law of Acceleration

Newton’s first law indicates that if there is no net force acting on an object at rest, it will remain at
rest; or if the object is moving, it continues moving with constant speed in a straight line. But what
happens if a net force is exerted on an object? That net force will cause acceleration.

The acceleration of an object is directly proportional to the net force acting on it,
and inversely proportional to the object’s mass. The direction of the acceleration
is in the direction of the net force acting on the object

Newton’s second law can be written as an equation:

∑F
a=
m
a more famous equation,

∑F = ma

Example 13.1 Force to accelerate a fast car

Estimate the net force needed to accelerate (a) a 1000 kg car at ½ g; (b) a 200 g apple at the
same rate.

Given: Solution:
ma = 1000 kg
mb = 200 g = 0.2 kg (a) The car’s acceleration is a = ½ g = ½ (9.8 m/s2) = 4.9 m/s2
a=½g
∑F = ma = (1000 kg) (4.9 m/s2) = 4900 N
RTF: ∑F (b) For the apple,
∑F = ma = (0.2 kg) (4.9 m/s2) = 0.98 N

SAN PEDRO RELOCATION CENTER NATIONAL HIGH SCHOOL – SCIENCE DEPARTMENT © 2020 73
Example 13.2 Force to stop a car

What average net force is required to bring a 1500-kg car to rest from a speed of 100km/h
within a distance of 55 m?

Given: Solution:
m = 1500 kg
vi = 100 km/h = 27.8 m/s Let’s calculate first for the acceleration using the kinematic
vf = 0 equations.
x = 55 m
v 2- v 2 (0)2 - (27.8m/s)2
a = 2(∆x)
f i
= = - 7 m/s2
RTF: ∑F 2(55 m)

The net force required is then

∑F = ma = (1500 kg) (-7 m/s2) = - 10,500 N


The force must be exerted in the opposite direction to the initial velocity, which is what the negative sign means.

Newton’s Third Law of Motion: The Law of Interaction

Whenever one object exerts a force on a second object, the second


object exerts an equal force in the opposite direction on the first

Take note that the forces here act on different objects. An example of Newton’s third law: when
an ice skater pushes against the wall, the wall pushes back and this force causes her to
accelerate. The action force is the force by the skater on the wall, the reaction force is the
force by the wall on the skater. The action and reaction force is exerted on two different
objects. Another example of Newton’s third law: the launch of a rocket. The rocket engine
pushes the gases downward, and the gases exert an equal and opposite force upward on the
rocket.

Figure 13.1 An example of Figure 13.2 An example of


Newton’s third law of motion: Newton’s third law of motion:
Ice skater pushes on a wall Rocket launch

74
SAN PEDRO RELOCATION CENTER NATIONAL HIGH SCHOOL – SCIENCE DEPARTMENT © 2020
FLASH CHECK
Read and answer the following questions,

1. You are waiting in line to use the diving board at your local pool. While
watching people dive into the pool from the board, you realize that using a
diving board to spring into the air before a dive is a good example of what
Law of motion? Explain your answer.
2. You dribble a basketball while walking on a basketball court. List and
describe at least 3 pairs of action-reaction forces in this situation.

3. Ben is being chased through the woods by a bull moose which he was
attempting to photograph. The enormous mass of the Bull Moose is
extremely intimidating. Yet, if Ben makes a zigzag pattern through the
woods, he will be able to use the large mass of the moose to his own
advantage. Explain this in terms of inertia and Newton's first law of motion.

4. What is the mass of an object that needs a force of 6 600 N to increase its
speed from rest to 107 m/s in 2.3 seconds?

QUICK REFERENCES
CONNECT https://www.straitstimes.com/asia/se-
asia/survival-guide-to-riding-the-jeepney-in-the-
philippines

https://macrovector.com/vector-images/grocery

https://www.insider.com/benefits-of-walking

https://www.businessinsider.com/spacex-falcon-
heavy-rocket-launch-live-video-youtube-2019-6

https://www.sas.upenn.edu/~kennethp/nkdievid2
.pdf

https://toonclips.com/design/66791

https://www.physicsclassroom.com/class/newtla
ws/Lesson-1/Newton-s-First-Law

Giancoli, D.C (2014). Physics, Principles with


Applications. 7th Edition. USA. Smith

SAN PEDRO RELOCATION CENTER NATIONAL HIGH SCHOOL – SCIENCE DEPARTMENT © 2020 75
WORKSHEET 7 – FORCES AND NEWTON’S LAWS OF MOTION

I. Read and answer the following conceptual questions.

1. A golf ball is hit with a golf club. While the ball flies through the air, which forces act on
the ball? Neglect air resistance. Explain your answer.
a. The force of the golf club acting on the ball
b. The force of gravity acting on the ball
c. The force of the ball acting forward through the air
d. All of the above
e. Both a and c

2. A school bus comes to a sudden stop, and all of the backpacks on the floor start to
slide forward. What force causes them to do that?

3. While driving down the road, a firefly strikes the windshield of a bus and makes a quite
obvious mess in front of the face of the driver. This is a clear case of Newton's third
law of motion. The firefly hit the bus and the bus hits the firefly. Which of the two
forces is greater: the force on the firefly or the force on the bus? Explain.

4. What makes a car go forward? Explain your answer.

II. Draw the free-body diagram of the following situations.

1. A girl is suspended motionless from a bar which hangs from the ceiling by two ropes.
2. An egg is free-falling from a nest in a tree. Neglect air resistance.
3. A skydiver is descending with a constant velocity. Consider air resistance

III. Free-body diagrams for four situations are shown below. For each situation,
determine the net force acting upon the object.

76
SAN PEDRO RELOCATION CENTER NATIONAL HIGH SCHOOL – SCIENCE DEPARTMENT © 2020
-
IV. Solve the following problems. Show your complete solution.

1. What force is needed to accelerate a sled (mass = 55 kg) at 1.4 m/s2 on a horizontal
frictionless ice?
2. What is the weight of a 68-kg astronaut (a) on Earth, (b) on the moon (g = 1.6 m/s2),
(c) on Mars (g = 3.71 m/s2), (d) in outer space traveling with constant velocity?
3. A 2.49 x 104 N Rolls-Royce Phantom traveling in the +x-direction makes an emergency
stop; the x-component of the net force acting on it is – 1.83 x104 N. What is its
acceleration?

V. Identify the action-reaction pairs on the following examples.

1. Action: Force of car on a bug; Reaction : ____________________________

2. Action: __________________; Reaction: Force of flower on hand.

3. Action: Force of finger on nose; Reaction: ____________________________

Give examples of action-reaction pairs found at home

1. _______________________________________________________________

2. _______________________________________________________________

3. _______________________________________________________________

4. _______________________________________________________________

5. _______________________________________________________________

REFERENCES:
http://www.reynolds.k12.pa.us/Downloads/free%20body%20worksheet.doc
https://www.sas.upenn.edu/~kennethp/nkdievid2.pdf
Giancoli, D.C (2014). Physics, Principles with Applications. 7th Edition. USA. Smith.

77
SAN PEDRO RELOCATION CENTER NATIONAL HIGH SCHOOL – SCIENCE DEPARTMENT © 2020
-
PHYSICS 1
Circular Motion

INSTANT TASK

BUCKET SPINNING
When you put a bucket of water upside down above your head, the water
will fall on you; this is what happens when we take a bath using a tabo. But
let’s see, through this activity, if it’s possible for the water to remain inside the
bucket even if it’s directly above you, upside down.

What you’ll need: Instructions:


**record a video of this activity and send it to me.
A bucket
Water
1. Fill the bucket until it is around half full with water.
2. Stand well clear of other people or anything else that could get in the
way.
3. Hold the bucket by its handle with your arm extended and start spinning
it by your side towards the sky and back to the ground in a circular
motion. The goal is to make the water stay inside the bucket.
4. Stop spinning before your arm gets tired, watching out for splashes as you
carefully bring the bucket back to rest on the ground.
Questions:

1. On your first attempt, what happens to the water inside the bucket?
2. What did you do in order to reach the goal?
3. Can you explain, in your own words, how is it possible for the water to
remain in a bucket?

78
SAN PEDRO RELOCATION CENTER NATIONAL HIGH SCHOOL – SCIENCE DEPARTMENT © 2020
DIRECT TALK

Circular Motion

When an object moves in a curved path, that object is experiencing a circular motion.
Circular motion could either be uniform or nonuniform. In this lesson we will focus only on Uniform
Circular Motion.

Uniform Circular Motion

An object that moves in a circle at constant speed is said to


experience uniform circular motion. The magnitude of the velocity
remains constant in this case, but the direction of the velocity
continuously changes as the object moves around the circle (Fig.
14.1). Because acceleration is defined as the rate of change of
velocity, a change in direction of velocity is an acceleration, just as a
change in magnitude is. Thus, an object revolving in a circle is
continuously accelerating, even when the speed remains constant.
This acceleration in a circle is called centripetal acceleration, aR. Figure 14.1 A small object moving in a
Centripetal means center-pointing or center-seeking, therefore, circle, showing how the velocity changes.
its direction is always towards the center. For the magnitude of the At each point, the instantaneous velocity
(velocity at that instant) is in a direction
centripetal acceleration, we use the equation
tangent to the circular path

v2
Where:
aR = v = speed
r r = radius
The direction of
motion and acceleration
Circular motion is often described in terms of frequency f, the number
are not the same direction,
of revolutions per second. The period T of an object revolving in a instead, their perpendicular
circle is the time required for one complete revolution. Period and to each other. (a v)
frequency are related by T = 1/f.
𝟐 𝐫
For the magnitude of velocity, we use the equation v=
𝐓

Example 14.1 Acceleration of a revolving ball

A 150 g ball at the end of a string is revolving uniformly in a horizontal circle of radius 0.600 m. The
ball makes 2.00 revolutions in a second. What is its centripetal force?

Given: Solution:
m = 150 g
r = 0.600 m Let’s identify first the speed of the ball
f = 2 rev/s
𝟐 𝐫 𝟐(𝟑.𝟏𝟒)(𝟎.𝟔𝟎𝟎 𝐦)
v= = = 7.54 m/s
𝐓 𝟎.𝟓 𝐬
RTF: aR
The centripetal acceleration is
m
v2 (7.54 )2
aR = = s
= 94.7 m/s2
r (0.600m)

SAN PEDRO RELOCATION CENTER NATIONAL HIGH SCHOOL – SCIENCE DEPARTMENT © 2020 79
According to Newton’s second law, an object that is accelerating must have a net force acting on
it. An object moving in a circle, such as a ball on the end of a string, must therefore have a force
applied to it to keep it moving in that circle. That is, a net force is necessary to give it centripetal
acceleration. This net force is also directed toward the center of the circle and is called
centripetal force FR. For the magnitude of the centripetal force, we use the equation

FR = maR

Example 14.2 Force on revolving ball (horizontal circle)

Estimate the force a person must exert on a string attached to a 0.150-kg ball to make the ball
revolve in a horizontal circle of radius 0.600 m. The ball makes 2.00 revolutions per second.
Ignore the string’s mass.
Solution:
Given:
m = 0.150 kg Let’s identify first the speed of the ball
r = 0.600 m 𝟐 𝐫 𝟐(𝟑.𝟏𝟒)(𝟎.𝟔𝟎𝟎 𝐦)
f = 2 rev/s v= = = 7.54 m/s
𝐓 𝟎.𝟓 𝐬

The centripetal Force is


RTF: FR
m
v2 (7.54 )2
FR = m = (0.150 kg)
s
= 14 N
r (0.600m)

Relative Velocity in One Dimension

You’ve no doubt observed how a car that is moving slowly forward appears to be moving
backward when you pass it, in general, when two observers measure the velocity of a moving
body, they get different results if one observer is moving relative to the other. The velocity seen
by a particular observer is called the velocity relative to that observer, or simply relative
velocity. In order to get the relative velocity, we simply get the difference between two
velocities. Let’s say for example we have the relative velocity v BA, this means that we are talking
about the velocity of object B with respect/relative to object A.

To calculate vBA, simply get the difference between vB and vA


When writing a subscript for
vBA = vB - vA relative velocity, the first subscript is
the object and the second subscript is
vB and vA means the velocity of object B and object A, the frame of reference

respectively, with respect to something that is stationary like the


Earth or someone that is not moving.

Example 14.3 Relative velocity on a straight road

You drive north on a straight two-lane road at a constant 88 km/h. A truck in the other lane
approaches you at a constant 104 km/h. Find (a) the truck’s velocity relative to you and (b)
your velocity relative to the truck.

Given: RTF:
vT = -104 km/h (a) vTY (b) vYT
vY = 88 km/h

80
SAN PEDRO RELOCATION CENTER NATIONAL HIGH SCHOOL – SCIENCE DEPARTMENT © 2020
Solution:

(a) vTY = vT - vY = (-104 km/h) - (88 km/h) = -192 km/h


(b) vYT = vY - vT = (88 km/h) - (-104 km/h) = +192 km/h

This means that for you, the truck is moving at 192 km/h in the opposite direction (south) and
for the truck, you are moving at 192km/h to the positive direction (north)

Example 14.4 Relative velocity on a moving train

A train travels at 60 m/s to the east with respect to the ground. A businessman on the train
runs at 5 m/s to the west with respect to the train. Find the velocity of the man with respect to
the ground.

Given: RTF:
vT = 60 m/s vM
vMT = -5 m/s

Solution:

vMT = vM - vT derive the formula to get vM


vM = vMT - vT
vM = (-5 m/s) - (60 m/s)
vM = 55 m/s

Relative Velocity in Two Dimension

When we talk about two dimensional motion, we mean that the motion has 2 directions that are
perpendicular to each other. Let’s consider this example.

Example 14.5 Flying in a crosswind

An airplane’s compass indicates that it is headed due north, and its airspeed indicator shows
that it is moving through the air at 240 km/h. If there is a 100-km/h wind from west to east,
what is the velocity of the airplane relative to the earth?

Given: RTF: As we can see, the diagram shows a right


vAW = 240 km/h vA triangle having vA as the hypotenuse. So in
vW = 100 m/s order to get vA, we use the Pythagorean
vW
theorem.
Solution:
In this problem, we take
vAW as the velocity of vA = (vAW )2 + (vW )2 = (240 )2 + (100)2
the airplane with respect
vA
to the wind, vW as the vAW
velocity of the wind with vA = 260 km/h
respect to the earth and 

vA as the velocity of the to get the angle ,


airplane with respect to
vW
the earth. = tan-l = tan-l (0.42) = 23°
vAW

SAN PEDRO RELOCATION CENTER NATIONAL HIGH SCHOOL – SCIENCE DEPARTMENT © 2020 81

You might also like